Bone Tumor Assessment
Quiz-summary
0 of 21 questions completed
Questions:
- 1
- 2
- 3
- 4
- 5
- 6
- 7
- 8
- 9
- 10
- 11
- 12
- 13
- 14
- 15
- 16
- 17
- 18
- 19
- 20
- 21
Information
This exam is not currently approved for CME: Just some fun learning!
Learning Objectives: At the conclusion of the general orthopedic self-assessment exam physician assistants should be familiar with radiographic characteristics of benign and malignant bone tumors, be able to identify common benign bone tumors including enchondroma, unicameral bone cyst, nonossifying fibroma, osteoid osteoma, aneurismal bone cyst, giant cell tumor, and osteochondroma, be able to identify common malignant bone tumors including osteosarcoma, chondrosarcoma, and Ewing’s sarcoma, and be able to identify and understand treatment of pathological fractures from metastatic disease.
You have already completed the quiz before. Hence you can not start it again.
Quiz is loading...
You must sign in or sign up to start the quiz.
You have to finish following quiz, to start this quiz:
Results
0 of 21 questions answered correctly
Your time:
Time has elapsed
You have reached 0 of 0 points, (0)
Average score |
|
Your score |
|
Categories
- Not categorized 0%
-
Congrats on completion!
- 1
- 2
- 3
- 4
- 5
- 6
- 7
- 8
- 9
- 10
- 11
- 12
- 13
- 14
- 15
- 16
- 17
- 18
- 19
- 20
- 21
- Answered
- Review
-
Question 1 of 21
1. Question
A 79-year old female presents with complaints of right hip pain for 2 months. She denies prior injury or precipitating event. The pain is made worse by lying on the affected side and occasionally hurts with hip motion. The pain is improved with NSAIDS and placing ice on the lateral aspect of her hip. Her primary care physician ordered x-rays which showed a bone lesion in the proximal femur. Her PCP was concerned she had bone cancer and ordered an MRI of the thigh to further evaluate. AP x-ray and a coronal MRI image are shown above. The patient denies a history of weight loss, fatigue, and history of cancer. On exam the patient has no pain with passive range of motion of the hip in all planes. She has pain with palpation over the greater trochanteric bursae. Motor and sensation are intact throughout the lower extremities. What is the most likely cause of this patient’s pain?
Correct
AP x-ray reveals a sclerotic bone lesion involving the proximal right femur. MRI reveals a chondroid type lesion with a sclerotic boarder consistent with an enchondroma. The patient’s physical exam findings are consistent with greater trochanteric bursitis. The enchondroma is an incidental finding.
An enchondroma is a benign tumor composed of hyaline cartilage located in the medullary cavity of bone. Enchondroma is the most common bone tumor found in the hands. The tumor may also be seen in the proximal femur, proximal humerus, tibia, and rarely, the spine and pelvis. The tumor is asymptomatic and generally found incidentally on x-rays. Large enchondromas of the hand may present with pathological fracture. X-ray characteristics include a radiolucent lesion located centrally in the medullary canal. The tumor calcifies over time revealing dots and arcs of calcification within the tumor.
A bone scan and MRI may be ordered to help differentiate from malignant transformation to a chondrosarcoma. Solitary enchodromas have less than a 1% chance of undergoing malignant transformation into a chondrosarcoma. Multiple enchondromas associated with Ollier’s disease (multiple endochondromatosis) and Maffucci Syndrome have a higher rate of malignant change. Findings that may be suspicious of malignant transformation include pain, increasing size, endosteal scalloping, and uptake on bone scan. Serial radiographs every 3-6 months for 1-2 years then annually are recommended to follow for possible changes. Biopsy followed by intralesional curettage and bone grafting may be indicated if malignant transformation is suspected or for large lesions prone to fracture.
ReferencesEnchondroma. http://www.bonetumor .org. Accessed June 11, 2015.
Enchondroma. http://www.orthobullets.com. Accessed June 11, 2015.
Lewis VO, Morris CD, Parsons TW. Malignant and benign bone tumors that you are likely to see. AAOS Instructional Course Lecture 2013; 62: 535-549.
Incorrect
AP x-ray reveals a sclerotic bone lesion involving the proximal right femur. MRI reveals a chondroid type lesion with a sclerotic boarder consistent with an enchondroma. The patient’s physical exam findings are consistent with greater trochanteric bursitis. The enchondroma is an incidental finding.
An enchondroma is a benign tumor composed of hyaline cartilage located in the medullary cavity of bone. Enchondroma is the most common bone tumor found in the hands. The tumor may also be seen in the proximal femur, proximal humerus, tibia, and rarely, the spine and pelvis. The tumor is asymptomatic and generally found incidentally on x-rays. Large enchondromas of the hand may present with pathological fracture. X-ray characteristics include a radiolucent lesion located centrally in the medullary canal. The tumor calcifies over time revealing dots and arcs of calcification within the tumor.
A bone scan and MRI may be ordered to help differentiate from malignant transformation to a chondrosarcoma. Solitary enchodromas have less than a 1% chance of undergoing malignant transformation into a chondrosarcoma. Multiple enchondromas associated with Ollier’s disease (multiple endochondromatosis) and Maffucci Syndrome have a higher rate of malignant change. Findings that may be suspicious of malignant transformation include pain, increasing size, endosteal scalloping, and uptake on bone scan. Serial radiographs every 3-6 months for 1-2 years then annually are recommended to follow for possible changes. Biopsy followed by intralesional curettage and bone grafting may be indicated if malignant transformation is suspected or for large lesions prone to fracture.
ReferencesEnchondroma. http://www.bonetumor .org. Accessed June 11, 2015.
Enchondroma. http://www.orthobullets.com. Accessed June 11, 2015.
Lewis VO, Morris CD, Parsons TW. Malignant and benign bone tumors that you are likely to see. AAOS Instructional Course Lecture 2013; 62: 535-549.
-
Question 2 of 21
2. Question
A 78-year old male presents with approximately 4 years of persistent right hip pain without a known injury. Initial x-rays of the hip, including an AP x-ray shown above, were taken shortly after the pain started. The x-ray showed a benign appearing lesion at the base of the femoral neck but was otherwise normal. He was advised to rest and use NSAIDS as needed. An MRI of the hip was ordered 2 years later during orthopedic follow-up for persistent pain. MRI showed trochanteric bursitis and a lobular 3 cm lesion at the base of the femoral neck. The lesion appeared benign and was diagnosed as a likely enchondroma. A steroid injection was performed for trochanteric bursitis which offered him only short term relief. The patient then saw a spine surgeon who thought the pain was related to his spinal stenosis. He received foraminal injections to the lumbar spine but they failed to relieve his hip and thigh pain. His pain is now starting to get intolerable with weight bearing and he is noticing more pain at night. Repeat AP x-ray of the hip appears to be unchanged from the AP x-ray done 4 years prior. What would be the next best step in this patient’s work-up?
Correct
It can often be difficult to determine whether a bone lesion is benign or malignant. As in this case, differentiating between a benign enchondroma and a malignant slow growing chondrosarcoma is the critical challenge. Treatment is vastly different between the two as enchondromas can be observed and chondrosarcomas require surgery. Identifying distinct clinical and radiographic features of bone lesions is essential in determining the treatment plan.
Determining where the patient’s pain is coming from is the initial and most important step in ruling out malignancy. Despite several treatments, the patient continued to complain of hip pain, including pain at night (hallmark sign of a malignancy). Repeat imaging can determine if the tumor changed over time which would be a sign of malignant transformation. Enchondromas are benign cartilaginous tumors that do not change in size and appearance over time. The patient’s repeat x-rays showed no cortical breakthrough, soft tissue mass, or periosteal reaction that would indicate the lesion was malignant. In this case, a repeat MRI would have the most value to compare to the prior MRI two years ago. Any subtle changes in the appearance of the lesion not appreciated on x-ray may be seen on MRI. Bone scans are less sensitive in the diagnostic work-up of bone lesions as they may show increased uptake for both benign and malignant lesions.
MRI findings suggestive of a chondrosaroma include endosteal scalloping involving two-thirds of the cortices depth, broken cortices, periosteal reaction, and the presence of a soft tissue mass. Clinical findings that suggest malignancy in this patient were worsening weight bearing pain and night pain. The patient’s pain failed to respond to other treatments as well suggesting the source of the pain was yet to be identified.
The patient later sustained a pathological fracture through the right femoral neck. CT scan with contrast of the chest, abdomen, and pelvis done for a metastatic work-up did not show any findings other than the isolated lesion at the femoral neck. A bone scan confirmed an isolated lesion at the right hip and a protein electrophoresis was negative. Intra-operative biopsies were taken during the hip fracture surgery and pathological findings showed a low grade chondrosaroma. The patient underwent a proximal femoral replacement with wide resection.
References
Jackson DW. Enchondroma vs. chondrosarcomas: tips for the orthopedic clinician. Orthopedics Today. December 2015.
Stanislavsky A, Gaillard F. Enchondroma vs. low grade sarcoma. http://www.radiopaedia.org. Accessed on 11/21/15.
Incorrect
It can often be difficult to determine whether a bone lesion is benign or malignant. As in this case, differentiating between a benign enchondroma and a malignant slow growing chondrosarcoma is the critical challenge. Treatment is vastly different between the two as enchondromas can be observed and chondrosarcomas require surgery. Identifying distinct clinical and radiographic features of bone lesions is essential in determining the treatment plan.
Determining where the patient’s pain is coming from is the initial and most important step in ruling out malignancy. Despite several treatments, the patient continued to complain of hip pain, including pain at night (hallmark sign of a malignancy). Repeat imaging can determine if the tumor changed over time which would be a sign of malignant transformation. Enchondromas are benign cartilaginous tumors that do not change in size and appearance over time. The patient’s repeat x-rays showed no cortical breakthrough, soft tissue mass, or periosteal reaction that would indicate the lesion was malignant. In this case, a repeat MRI would have the most value to compare to the prior MRI two years ago. Any subtle changes in the appearance of the lesion not appreciated on x-ray may be seen on MRI. Bone scans are less sensitive in the diagnostic work-up of bone lesions as they may show increased uptake for both benign and malignant lesions.
MRI findings suggestive of a chondrosaroma include endosteal scalloping involving two-thirds of the cortices depth, broken cortices, periosteal reaction, and the presence of a soft tissue mass. Clinical findings that suggest malignancy in this patient were worsening weight bearing pain and night pain. The patient’s pain failed to respond to other treatments as well suggesting the source of the pain was yet to be identified.
The patient later sustained a pathological fracture through the right femoral neck. CT scan with contrast of the chest, abdomen, and pelvis done for a metastatic work-up did not show any findings other than the isolated lesion at the femoral neck. A bone scan confirmed an isolated lesion at the right hip and a protein electrophoresis was negative. Intra-operative biopsies were taken during the hip fracture surgery and pathological findings showed a low grade chondrosaroma. The patient underwent a proximal femoral replacement with wide resection.
References
Jackson DW. Enchondroma vs. chondrosarcomas: tips for the orthopedic clinician. Orthopedics Today. December 2015.
Stanislavsky A, Gaillard F. Enchondroma vs. low grade sarcoma. http://www.radiopaedia.org. Accessed on 11/21/15.
-
Question 3 of 21
3. Question
A 9-year old girl presents to the office with right arm pain after falling down a few steps at home. She felt her arm twist as she grabbed the rail to catch her fall. On x-ray a pathological fracture of the proximal humerus is found. The patient denies any pain in the arm prior to the fall. What is the best treatment option?
Correct
The patient has a pathological fracture, defined as a fracture that occurs as a result of abnormal bone. The patient’s bone has been weakened by a large unicameral bone cyst seen on the x-ray above. A unicameral bone cyst (UBC), often referred to as a simple bone cyst, is a benign lesion that is most commonly found in the proximal humerus of young patients. UBC’s are asymptomatic and found incidentally on x-ray or after a pathological fracture has occurred. UBC’s are most commonly found in patients between the ages of 4-10 years old and rarely found in patients over 20 years old. The typical location is central and metaphyseal. X-rays will show a well marginated, symmetrical, radiolucent lesion that may be unilocular or septated.
UBC’s are divided into two groups, latent cysts and active cysts, which helps guide treatment. UBC’s are considered active if adjacent to the physis. Characteristically, an active cyst will develop in patients under 10 years old, continue to enlarge during observation, and are surrounded by a thin cortical shell (more likely to cause fracture). Latent or passive cysts will grow away from the physis and have normal bone present between the cyst and physis. A latent cyst also ceases to expand, occurs in patients over 12 years old, occasionally shows ossification, and is less likely to fracture.
A “fallen leaf” sign is a pathologic fracture with a fallen cortical fragment in the base of the empty cyst. This finding is pathognomonic for a UBC. First line treatment for minimally displaced pathological fractures through UBC’s of the proximal humerus is sling immobilization for a period of 6 weeks. Fracture recurrence is common particularly in patients with a lesion that occupies greater than 85% of the transverse diameter of the bone and if the width of the cortical wall is less than .5 mm thick. The fracture will heal within in 6 weeks, and in 15% of cases, native bone will fill the cyst after the fracture heals.
The natural history of UBC’s is to heal with skeletal maturity so older patients (older than 10) are more likely to heal before a fracture occurs. UBC’s found in younger patients are more likely to fracture over time and often require prophylactic treatment. Treatment with serial intralesional injections of methylprednisolone is indicated in active cysts that communicate with the physis. Intralesional steroid injections require general anesthesia and usually take 3-4 separate procedures before the cyst completely heals. An inflow catheter and outflow catheter is inserted into the bone for irrigation prior to the steroid injection. Open curettage and bone grafting of lesions in the proximal humerus is less commonly performed due to the morbidity of surgery, but reserved for patients if not responsive to steroid injections. Recurrence rate after curettage and bone grafting is approximately 50% for active cysts and 10% for latent cysts. Treatment of pathological fractures through the proximal femur includes curettage and bone grafting with internal fixation due to the high rate of refracture and malunion.
The x-ray below shows the UBC at the time of fracture and 6 months later. The fracture stimulated ossification of the lesion.
References
Simple bone cyst (unicameral) http://www.wheelessonline.com. Accessed on 4/30/15.
Dormans JP, Pill SG. Fractures through bone cysts: unicameral bone cysts, aneurysmal bone cysts, fibrous cortical defects, and nonossifing fibromas. AAOS Instructional Course Lectures. Volume 51, 2002.
Unicameral Bone Cysts: General Characteristics and Management Controversies. JAAOS. May 2014. (22) 5. 295-303.
Incorrect
The patient has a pathological fracture, defined as a fracture that occurs as a result of abnormal bone. The patient’s bone has been weakened by a large unicameral bone cyst seen on the x-ray above. A unicameral bone cyst (UBC), often referred to as a simple bone cyst, is a benign lesion that is most commonly found in the proximal humerus of young patients. UBC’s are asymptomatic and found incidentally on x-ray or after a pathological fracture has occurred. UBC’s are most commonly found in patients between the ages of 4-10 years old and rarely found in patients over 20 years old. The typical location is central and metaphyseal. X-rays will show a well marginated, symmetrical, radiolucent lesion that may be unilocular or septated.
UBC’s are divided into two groups, latent cysts and active cysts, which helps guide treatment. UBC’s are considered active if adjacent to the physis. Characteristically, an active cyst will develop in patients under 10 years old, continue to enlarge during observation, and are surrounded by a thin cortical shell (more likely to cause fracture). Latent or passive cysts will grow away from the physis and have normal bone present between the cyst and physis. A latent cyst also ceases to expand, occurs in patients over 12 years old, occasionally shows ossification, and is less likely to fracture.
A “fallen leaf” sign is a pathologic fracture with a fallen cortical fragment in the base of the empty cyst. This finding is pathognomonic for a UBC. First line treatment for minimally displaced pathological fractures through UBC’s of the proximal humerus is sling immobilization for a period of 6 weeks. Fracture recurrence is common particularly in patients with a lesion that occupies greater than 85% of the transverse diameter of the bone and if the width of the cortical wall is less than .5 mm thick. The fracture will heal within in 6 weeks, and in 15% of cases, native bone will fill the cyst after the fracture heals.
The natural history of UBC’s is to heal with skeletal maturity so older patients (older than 10) are more likely to heal before a fracture occurs. UBC’s found in younger patients are more likely to fracture over time and often require prophylactic treatment. Treatment with serial intralesional injections of methylprednisolone is indicated in active cysts that communicate with the physis. Intralesional steroid injections require general anesthesia and usually take 3-4 separate procedures before the cyst completely heals. An inflow catheter and outflow catheter is inserted into the bone for irrigation prior to the steroid injection. Open curettage and bone grafting of lesions in the proximal humerus is less commonly performed due to the morbidity of surgery, but reserved for patients if not responsive to steroid injections. Recurrence rate after curettage and bone grafting is approximately 50% for active cysts and 10% for latent cysts. Treatment of pathological fractures through the proximal femur includes curettage and bone grafting with internal fixation due to the high rate of refracture and malunion.
The x-ray below shows the UBC at the time of fracture and 6 months later. The fracture stimulated ossification of the lesion.
References
Simple bone cyst (unicameral) http://www.wheelessonline.com. Accessed on 4/30/15.
Dormans JP, Pill SG. Fractures through bone cysts: unicameral bone cysts, aneurysmal bone cysts, fibrous cortical defects, and nonossifing fibromas. AAOS Instructional Course Lectures. Volume 51, 2002.
Unicameral Bone Cysts: General Characteristics and Management Controversies. JAAOS. May 2014. (22) 5. 295-303.
-
Question 4 of 21
4. Question
Which choice below correctly matches the bone tumor with its most common location of occurrence?
Correct
The location of a bone tumor within the skeleton can aid in establishing a diagnosis. The tumors location within the bone is often described using the following: diaphyseal, metaphyseal, or epiphyseal; eccentric or central; and medullary vs. periosteal. Commonly seen epiphyseal tumors include chondroblastoma, giant cell tumor, and aneurysmal bone cysts. Metaphyseal tumors include non-ossifying fibroma (NOF) near the growth plate, and bone cysts (solitary, aneurysmal, and giant cell). Osteochondroma, chondrosarcoma, osteogenic sarcoma, and the juvenile Brodie abscess also form in this area. In the diaphysis of bone, fibrous dysplasia, ostoeblastoma, Ewing sarcoma, and osteoid osteoma should be suspected.
In the transverse or AP plane, the location can be described based on the midline of the bone. Lesions arising in the center include enchondromas, lesions that are eccentric include giant cell tumors and osteosarcomas, and lesions that are cortical include osteoid osteomas and non-ossifying fibromas.
Some lesions may seem to sit on the bone cortex or periosteum (parosteal), such as osteochondroma or parosteal osteosarcoma. A list of bone lesions with their commonest location can be found below. As you can see, the most common location for all malignant and benign tumors is around the knee.
Benign lesions
Simple bone cyst: proximal humerus, proximal femur
Osteoid Osteoma: proximal tibia, distal femur
Aneurysmal bone cyst, giant cell tumor, and osteosarcoma: distal femur, proximal tibia Enchondroma: metaphysis of small bones of hands and feet
Fibrous dysplasia: ribs, proximal femur, tibia, and distal femur
Osteochondroma: distal femur-proximal tibia, proximal humerus
Osteoid osteoma: femur, tibiaMalignant lesions
Osteosarcoma: femur, tibia, humerus, fibula, ileum
Chondrosarcoma: pelvis, proximal femur
Ewing’s: femur, fibula-tibia
Chondroblastoma: pelvis, femur
Osteoblastoma: posterior spine
Myeloma: vertebra.
Metastases: vertebrae, ribs, pelvis, femur, humerus
References
Hameed, M., & Dorfman, H. (2011). Primary malignant bone tumors-recent developments. Seminars in Diagnostic Pathology, 28, 86-101. http://dx.doi.org/10.1053j.semdp.2011.02.002
Lucent lesions of bone. (2011). Retrieved from http://www.rad.washington.edu/academics/academic-sections/msk/teaching-materials/online-musculoskeletal-radiology-book/lucent-lesions-of-boneSabat, MD, D. (2010). Radiology of bone tumors [PowerPoint slides]. Retrieved from Google Scholar/Slideshare: http://www.slideshare.net/
Sanders, T. G., & Parsons, T. W. (2001). Radiographic imaging of musculoskeletal neoplasia. Cancer Control, 8, 221-230. Retrieved from http://www.medscape.com/viewarticle/409049
Incorrect
The location of a bone tumor within the skeleton can aid in establishing a diagnosis. The tumors location within the bone is often described using the following: diaphyseal, metaphyseal, or epiphyseal; eccentric or central; and medullary vs. periosteal. Commonly seen epiphyseal tumors include chondroblastoma, giant cell tumor, and aneurysmal bone cysts. Metaphyseal tumors include non-ossifying fibroma (NOF) near the growth plate, and bone cysts (solitary, aneurysmal, and giant cell). Osteochondroma, chondrosarcoma, osteogenic sarcoma, and the juvenile Brodie abscess also form in this area. In the diaphysis of bone, fibrous dysplasia, ostoeblastoma, Ewing sarcoma, and osteoid osteoma should be suspected.
In the transverse or AP plane, the location can be described based on the midline of the bone. Lesions arising in the center include enchondromas, lesions that are eccentric include giant cell tumors and osteosarcomas, and lesions that are cortical include osteoid osteomas and non-ossifying fibromas.
Some lesions may seem to sit on the bone cortex or periosteum (parosteal), such as osteochondroma or parosteal osteosarcoma. A list of bone lesions with their commonest location can be found below. As you can see, the most common location for all malignant and benign tumors is around the knee.
Benign lesions
Simple bone cyst: proximal humerus, proximal femur
Osteoid Osteoma: proximal tibia, distal femur
Aneurysmal bone cyst, giant cell tumor, and osteosarcoma: distal femur, proximal tibia Enchondroma: metaphysis of small bones of hands and feet
Fibrous dysplasia: ribs, proximal femur, tibia, and distal femur
Osteochondroma: distal femur-proximal tibia, proximal humerus
Osteoid osteoma: femur, tibiaMalignant lesions
Osteosarcoma: femur, tibia, humerus, fibula, ileum
Chondrosarcoma: pelvis, proximal femur
Ewing’s: femur, fibula-tibia
Chondroblastoma: pelvis, femur
Osteoblastoma: posterior spine
Myeloma: vertebra.
Metastases: vertebrae, ribs, pelvis, femur, humerus
References
Hameed, M., & Dorfman, H. (2011). Primary malignant bone tumors-recent developments. Seminars in Diagnostic Pathology, 28, 86-101. http://dx.doi.org/10.1053j.semdp.2011.02.002
Lucent lesions of bone. (2011). Retrieved from http://www.rad.washington.edu/academics/academic-sections/msk/teaching-materials/online-musculoskeletal-radiology-book/lucent-lesions-of-boneSabat, MD, D. (2010). Radiology of bone tumors [PowerPoint slides]. Retrieved from Google Scholar/Slideshare: http://www.slideshare.net/
Sanders, T. G., & Parsons, T. W. (2001). Radiographic imaging of musculoskeletal neoplasia. Cancer Control, 8, 221-230. Retrieved from http://www.medscape.com/viewarticle/409049
-
Question 5 of 21
5. Question
A 16-year old male presents to your office with anterior medial knee pain after being hit during a hockey game. He denies having knee pain before this injury. On exam of the knee you notice subtle laxity with valgus stress consistent with an MCL sprain. The rest of the exam is unremarkable. When reviewing x-rays of the patient you notice a well circumscribed radiolucent lesion on the lateral side of the proximal tibia. What is the name of this bone lesion?
Correct
Differentiating benign and malignant bone tumors is a crucial skill to possess in orthopedic practice. Being able to identify benign tumors accurately prevents unnecessary imaging and referral. Most benign tumors are asymptomatic and found incidentally on x-ray. However, aggressive benign bone tumors may cause bone destruction and pain. Benign bone tumors are commonly differentiated by size, location, symptoms, x-ray appearance, and patient age. Benign tumors can be accurately diagnosed by x-ray in 90% of patients. The biological activity of the tumor is determined by the margin between the normal bone and the tumor. A narrow, clearly delineated margin suggests a benign process. When the transition zone is unclear, or it has a moth eaten pattern of destruction, an aggressive benign tumor or malignancy should be suspected.
Non-ossifying fibroma, or a fibrous cortical defect, is the most common tumor in childhood and is estimated to be present in 30% of all children and adolescences. Fibrous cortical defect and non-ossifying fibroma are histologically identical and commonly used interchangeably. However, fibrous cortical defects are smaller and confined to the cortex where non-ossifying fibromas involve the metaphysis. Common locations include the metaphysis of long bones, particularly in the distal femur, proximal tibia, and distal tibia. Over 80% occur in the lower extremity. The lesion is asymptomatic and typically found incidentally on x-ray. X-rays will show a radiolucent, eccentrically located, multilocular appearance that is surrounded by a sclerotic rim of bone. The lesion expands toward the diaphysis with growth of the bone during childhood. Most lesions will regress during skeletal maturity and ossify into adulthood.
Diagnosis is most commonly made by x-ray alone and addition tests, including MRI or biopsy, are rarely necessary. The characteristic radiographic appearance of metaphyseal location, clearly defined cortex, well defined margins, maximum size of 6-7 cm, and found incidentally during skeletal growth are distinct features that should be observed.
Because most NOF’s will heal with time, conservative treatment with observation is the first line treatment. Patient should be advised that the lesion should remain asymptomatic, ossify during skeletal maturity, and that progression to malignancy has not been shown. Lesions located in high stress areas, composed of 50% or more of the bone diameter, and have thin cortices are at a high risk of pathological fracture. A CT may be helpful to predict fracture risk by to determining cortical thickness. Curettage and bone grafting may be indicated in large symptomatic lesions at risk for fracture, particularly in children who are very active or play sports. Pathological fracture resulting from an NOF can be treated with closed treatment in most cases.
References
Non-ossifying fibroma. http://www.orthobullets.com. Accessed on 4/3/15.
Temple HT, Scully SP, Aboulafia AJ. Benign bone tumors. AAOS Instructional Course Lectures 2002. Volume 51, 429-439.
Incorrect
Differentiating benign and malignant bone tumors is a crucial skill to possess in orthopedic practice. Being able to identify benign tumors accurately prevents unnecessary imaging and referral. Most benign tumors are asymptomatic and found incidentally on x-ray. However, aggressive benign bone tumors may cause bone destruction and pain. Benign bone tumors are commonly differentiated by size, location, symptoms, x-ray appearance, and patient age. Benign tumors can be accurately diagnosed by x-ray in 90% of patients. The biological activity of the tumor is determined by the margin between the normal bone and the tumor. A narrow, clearly delineated margin suggests a benign process. When the transition zone is unclear, or it has a moth eaten pattern of destruction, an aggressive benign tumor or malignancy should be suspected.
Non-ossifying fibroma, or a fibrous cortical defect, is the most common tumor in childhood and is estimated to be present in 30% of all children and adolescences. Fibrous cortical defect and non-ossifying fibroma are histologically identical and commonly used interchangeably. However, fibrous cortical defects are smaller and confined to the cortex where non-ossifying fibromas involve the metaphysis. Common locations include the metaphysis of long bones, particularly in the distal femur, proximal tibia, and distal tibia. Over 80% occur in the lower extremity. The lesion is asymptomatic and typically found incidentally on x-ray. X-rays will show a radiolucent, eccentrically located, multilocular appearance that is surrounded by a sclerotic rim of bone. The lesion expands toward the diaphysis with growth of the bone during childhood. Most lesions will regress during skeletal maturity and ossify into adulthood.
Diagnosis is most commonly made by x-ray alone and addition tests, including MRI or biopsy, are rarely necessary. The characteristic radiographic appearance of metaphyseal location, clearly defined cortex, well defined margins, maximum size of 6-7 cm, and found incidentally during skeletal growth are distinct features that should be observed.
Because most NOF’s will heal with time, conservative treatment with observation is the first line treatment. Patient should be advised that the lesion should remain asymptomatic, ossify during skeletal maturity, and that progression to malignancy has not been shown. Lesions located in high stress areas, composed of 50% or more of the bone diameter, and have thin cortices are at a high risk of pathological fracture. A CT may be helpful to predict fracture risk by to determining cortical thickness. Curettage and bone grafting may be indicated in large symptomatic lesions at risk for fracture, particularly in children who are very active or play sports. Pathological fracture resulting from an NOF can be treated with closed treatment in most cases.
References
Non-ossifying fibroma. http://www.orthobullets.com. Accessed on 4/3/15.
Temple HT, Scully SP, Aboulafia AJ. Benign bone tumors. AAOS Instructional Course Lectures 2002. Volume 51, 429-439.
-
Question 6 of 21
6. Question
A 5 year old African American boy presents to the orthopedic clinic complaining of right knee pain. The mother explains that the child has been complaining about the right knee pain for about a month and he started limping a few days ago. She does not recall any trauma or injuries that may have caused this. She has tried an ace wrap and hot packs on the knee, but nothing has helped alleviate the pain. She feels that the pain is increasingly getting worse. She denies any fevers or recent illnesses. She admits that his appetite has not been great and she thinks he has lost some weight. She reports that he is otherwise healthy, on no medications and has no known drug allergies. His immunizations are up to date. On physical exam of the right knee, there is mild swelling around the proximal medial tibia, which is firm to palpation. There is increased temperature and tenderness in the area without effusion. There is full range of motion and no instability. Examination of the left knee is unremarkable. AP and lateral x rays of the right knee, tibia, and fibula are obtained and demonstrate a lucent lesion in the proximal tibia metaphysis measuring approximately 2.5 cm in length by 2.0 cm in width. There is no periosteal reaction. There is a subtle transverse lucent line extending from the lesion medially for a length of about 1.5 cm. There appears to be some soft tissue swelling posteriorly and medially. Based on these findings, which of the following is the most likely diagnosis?Correct
The differential diagnosis for this finding includes osteomyelitis and a bone tumor. Lymphoma and multiple myeloma are unlikely because these are usually diaphyseal. Ewing sarcoma usually has periosteal reaction, which is not evident on this film. There is no calcification within the lesion, which us usually seen in a chondrosarcoma. Furthermore, this lesion is not as well defined as would be expected for a giant cell tumor or a eosinophilic granuloma.
The patient had an MRI which further demonstrated the lesion and the patient was referred to an orthopedic oncologist for evaluation. The patient was diagnosed with osteosarcoma and had an amputation. He was treated with chemotherapy prior to the amputation and for the year following. He is now 15 years old and is cancer free.
Osteosarcoma is the sixth most common malignancy in childhood. The peak occurrence is usually during the adolescent growth spurt and suggests a relationship between rapid bone growth and malignant transformation. This type of bone tumor occurs most frequently at sites where the greatest increase in length and size of bone occurs. The metaphyses of long tubular bones are primarily affected, with the distal femur accounting for more than 40% of cases. The most common presenting symptom is pain over the involved area with or without soft tissue mass. Usually the patient has had the pain for several months prior to diagnosis and systemic symptoms are rare. The patient may have an elevated serum alkaline phosphatase or LDH levels. X ray findings show destruction of the normal bony trabecular pattern with indistinct margins. There is also periosteal new bone formation. A soft tissue mass is frequently noted as well. MRI is more sensitive in defining the extent of the primary tumor. The most common sites of metastases are the lung and additional boney sites. A tissue sample is needed for confirmation of the diagnosis. The surgeon who will be carrying out any additional surgical procedures should perform the biopsy because the placement of the incision for biopsy is important. Historically, patients receiving surgery alone developed pulmonary metastases within 6 months of surgery, however, with adjuvant chemotherapy, disease free survival rates increased in patients followed for 3-10 years. Survival rates were shown to improve when chemotherapy was initiated prior to the surgery and continued for one year following. Relapses beyond 3 years are unusual. Patients with localized disease with 90% tumor necrosis have a 70-85% long-term, disease-free survival rate.
References
Graham DK, Craddock JA, Quinones RR, Keating AK, Maloney K, Foreman NK, Giller RH, Greffe BS. Neoplastic Disease. In: Hay WW, Jr., Levin MJ, Deterding RR, Abzug MJ. eds. CURRENT Diagnosis & Treatment: Pediatrics, 22e. New York, NY: McGraw-Hill; 2013. http://accessmedicine.mhmedical.com/content.aspx?bookid=1016&Sectionid=61604063. Accessed April 22, 2015.Miner Haygood T, Sayyouh MH. Chapter 6. Musculoskeletal Imaging. In: Chen MM, Pope TL, Ott DJ. eds. Basic Radiology, 2e. New York, NY: McGraw-Hill; 2011. http://accessmedicine.mhmedical.com/content.aspx?bookid=360&Sectionid=39669014. Accessed April 22, 2015.
Incorrect
The differential diagnosis for this finding includes osteomyelitis and a bone tumor. Lymphoma and multiple myeloma are unlikely because these are usually diaphyseal. Ewing sarcoma usually has periosteal reaction, which is not evident on this film. There is no calcification within the lesion, which us usually seen in a chondrosarcoma. Furthermore, this lesion is not as well defined as would be expected for a giant cell tumor or a eosinophilic granuloma.
The patient had an MRI which further demonstrated the lesion and the patient was referred to an orthopedic oncologist for evaluation. The patient was diagnosed with osteosarcoma and had an amputation. He was treated with chemotherapy prior to the amputation and for the year following. He is now 15 years old and is cancer free.
Osteosarcoma is the sixth most common malignancy in childhood. The peak occurrence is usually during the adolescent growth spurt and suggests a relationship between rapid bone growth and malignant transformation. This type of bone tumor occurs most frequently at sites where the greatest increase in length and size of bone occurs. The metaphyses of long tubular bones are primarily affected, with the distal femur accounting for more than 40% of cases. The most common presenting symptom is pain over the involved area with or without soft tissue mass. Usually the patient has had the pain for several months prior to diagnosis and systemic symptoms are rare. The patient may have an elevated serum alkaline phosphatase or LDH levels. X ray findings show destruction of the normal bony trabecular pattern with indistinct margins. There is also periosteal new bone formation. A soft tissue mass is frequently noted as well. MRI is more sensitive in defining the extent of the primary tumor. The most common sites of metastases are the lung and additional boney sites. A tissue sample is needed for confirmation of the diagnosis. The surgeon who will be carrying out any additional surgical procedures should perform the biopsy because the placement of the incision for biopsy is important. Historically, patients receiving surgery alone developed pulmonary metastases within 6 months of surgery, however, with adjuvant chemotherapy, disease free survival rates increased in patients followed for 3-10 years. Survival rates were shown to improve when chemotherapy was initiated prior to the surgery and continued for one year following. Relapses beyond 3 years are unusual. Patients with localized disease with 90% tumor necrosis have a 70-85% long-term, disease-free survival rate.
References
Graham DK, Craddock JA, Quinones RR, Keating AK, Maloney K, Foreman NK, Giller RH, Greffe BS. Neoplastic Disease. In: Hay WW, Jr., Levin MJ, Deterding RR, Abzug MJ. eds. CURRENT Diagnosis & Treatment: Pediatrics, 22e. New York, NY: McGraw-Hill; 2013. http://accessmedicine.mhmedical.com/content.aspx?bookid=1016&Sectionid=61604063. Accessed April 22, 2015.Miner Haygood T, Sayyouh MH. Chapter 6. Musculoskeletal Imaging. In: Chen MM, Pope TL, Ott DJ. eds. Basic Radiology, 2e. New York, NY: McGraw-Hill; 2011. http://accessmedicine.mhmedical.com/content.aspx?bookid=360&Sectionid=39669014. Accessed April 22, 2015.
-
Question 7 of 21
7. Question
Which x-ray finding below would be suggestive of a malignant bone tumor?
Correct
Bone destruction can be lytic or sclerotic. Benign appearing lesions often have a well-defined margin with a short zone of transition between the lesion and the surrounding bone. A permeative lesion merges with the uninvolved bone and has a long zone of transition. Finally, moth-eaten lesions have areas of destruction with ragged margins, less defined lesion margin and a long zone of transition. The radiographic appearance of the margin tends to correspond well with the aggressiveness of the tumor.
Margins between the lesion and the bone can indicate the aggressiveness of the lesion. Slow progressing lesions are “walled-off” by native bone, producing well defined, distinct margins. Lesions that progress rapidly destroy bone producing indistinct margins.
Sclerotic margins suggest a benign tumor including unicameral bone cysts (UBC), enchondroma, fibrous dysplasia (FD), chondroblastoma, and giant cell tumors (GCT). Well defined but non-sclerotic margins are seen in GCT, enchondroma, chondroblastoma, FD, and chondrosarcoma. Lytic lesions with ill-defined margins are found in chondrosarcoma, osteosarcoma, lymphoma, metastasis, GCT and infections. Moth-eaten lesions are seen in myeloma, infection, osteosarcoma, chondrosarcoma and lymphoma. Permeative (poorly demarcated) lesions are found in Ewing’s, myeloma metastasis, lymphoma, and osteosarcoma.
Age is an important factor in determining whether a bone tumor is likely to be malignant or benign. The majority of benign bone tumors occur in patients less than 30 years old. Bone tumors in patients over 30 years old suggest malignancy including metastatic disease, sarcoma, myeloma, and lymphoma.
Bone has a limited number of reactions that it can exhibit in response to a tumor. Bone can be destroyed (lysis) leaving a lytic lesion, react to the advancing tumor causing sclerosis, and lastly bone can remodel in the face of the tumor, known as periosteal reaction. This reaction is predicated by the rate of tumor growth. Rapid growth shows destruction or lysis and slow growth shows a predominance of sclerosis.
Periosteal reaction has to mineralize before it shows on plain films. This process can take 10 to 21 days and is dependent on the aggressiveness and duration of the tumor. Thick uninterrupted periosteal reaction suggests a long standing benign process. Spiculated or lamellated periosteal reaction is suggestive of an aggressive process and most likely a tumor. This process can be seen in what is known as a Codman Triangle. This process shows periosteal reaction and elevation, with the advancing tumor destroying this as its margin advances.
In summary, radiographic features of a benign tumor include distinct cortical margins, well demarcated sclerotic boarders, thick or solid periosteal reaction, and no soft tissue mass. Radiographic features of a malignant process include cortical destruction, aggressive irregular periosteal reaction including a sunburst pattern, onion peeling, Codman’s triangle, and soft tissue involvement.
References
Hameed, M., & Dorfman, H. (2011). Primary malignant bone tumors-recent developments. Seminars in Diagnostic Pathology, 28, 86-101. http://dx.doi.org/10.1053j.semdp.2011.02.002
Lucent lesions of bone. (2011). Retrieved from http://www.rad.washington.edu/academics/academic-sections/msk/teaching-materials/online-musculoskeletal-radiology-book/lucent-lesions-of-boneSabat, MD, D. (2010). Radiology of bone tumors [PowerPoint slides]. Retrieved from Google Scholar/Slideshare: http://www.slideshare.net/
Sanders, T. G., & Parsons, T. W. (2001). Radiographic imaging of musculoskeletal neoplasia. Cancer Control, 8, 221-230. Retrieved from http://www.medscape.com/viewarticle/409049
Incorrect
Bone destruction can be lytic or sclerotic. Benign appearing lesions often have a well-defined margin with a short zone of transition between the lesion and the surrounding bone. A permeative lesion merges with the uninvolved bone and has a long zone of transition. Finally, moth-eaten lesions have areas of destruction with ragged margins, less defined lesion margin and a long zone of transition. The radiographic appearance of the margin tends to correspond well with the aggressiveness of the tumor.
Margins between the lesion and the bone can indicate the aggressiveness of the lesion. Slow progressing lesions are “walled-off” by native bone, producing well defined, distinct margins. Lesions that progress rapidly destroy bone producing indistinct margins.
Sclerotic margins suggest a benign tumor including unicameral bone cysts (UBC), enchondroma, fibrous dysplasia (FD), chondroblastoma, and giant cell tumors (GCT). Well defined but non-sclerotic margins are seen in GCT, enchondroma, chondroblastoma, FD, and chondrosarcoma. Lytic lesions with ill-defined margins are found in chondrosarcoma, osteosarcoma, lymphoma, metastasis, GCT and infections. Moth-eaten lesions are seen in myeloma, infection, osteosarcoma, chondrosarcoma and lymphoma. Permeative (poorly demarcated) lesions are found in Ewing’s, myeloma metastasis, lymphoma, and osteosarcoma.
Age is an important factor in determining whether a bone tumor is likely to be malignant or benign. The majority of benign bone tumors occur in patients less than 30 years old. Bone tumors in patients over 30 years old suggest malignancy including metastatic disease, sarcoma, myeloma, and lymphoma.
Bone has a limited number of reactions that it can exhibit in response to a tumor. Bone can be destroyed (lysis) leaving a lytic lesion, react to the advancing tumor causing sclerosis, and lastly bone can remodel in the face of the tumor, known as periosteal reaction. This reaction is predicated by the rate of tumor growth. Rapid growth shows destruction or lysis and slow growth shows a predominance of sclerosis.
Periosteal reaction has to mineralize before it shows on plain films. This process can take 10 to 21 days and is dependent on the aggressiveness and duration of the tumor. Thick uninterrupted periosteal reaction suggests a long standing benign process. Spiculated or lamellated periosteal reaction is suggestive of an aggressive process and most likely a tumor. This process can be seen in what is known as a Codman Triangle. This process shows periosteal reaction and elevation, with the advancing tumor destroying this as its margin advances.
In summary, radiographic features of a benign tumor include distinct cortical margins, well demarcated sclerotic boarders, thick or solid periosteal reaction, and no soft tissue mass. Radiographic features of a malignant process include cortical destruction, aggressive irregular periosteal reaction including a sunburst pattern, onion peeling, Codman’s triangle, and soft tissue involvement.
References
Hameed, M., & Dorfman, H. (2011). Primary malignant bone tumors-recent developments. Seminars in Diagnostic Pathology, 28, 86-101. http://dx.doi.org/10.1053j.semdp.2011.02.002
Lucent lesions of bone. (2011). Retrieved from http://www.rad.washington.edu/academics/academic-sections/msk/teaching-materials/online-musculoskeletal-radiology-book/lucent-lesions-of-boneSabat, MD, D. (2010). Radiology of bone tumors [PowerPoint slides]. Retrieved from Google Scholar/Slideshare: http://www.slideshare.net/
Sanders, T. G., & Parsons, T. W. (2001). Radiographic imaging of musculoskeletal neoplasia. Cancer Control, 8, 221-230. Retrieved from http://www.medscape.com/viewarticle/409049
-
Question 8 of 21
8. Question
A 10-year old female presents to your office with a 2 day history of left ankle pain after a twisting injury during a field hockey game. She presented to an urgent care office the day of injury where x-rays showed a large bone lesion in the calcaneus. No acute fractures were seen. She was diagnosed with a lateral ankle sprain, splinted, and sent for orthopedic follow-up. The patient denies having ankle pain prior to the recent injury. On exam the patient has tenderness over the lateral anterior talofibular ligament consistent with a lateral ankle sprain. The rest of the exam is unremarkable. When reviewing the patient’s ankle x-rays you notice a large, round, well circumscribed radiolucent lesion in calcaneus. A lateral x-ray of the calcaneus and a sagittal T1 MRI image are shown above. What is the most likely diagnosis?
Correct
Osteoid osteomas, aneurysmal bone cysts, non-ossfiying fibromas, unicameral bone cysts, and giant cell tumors are all benign bone tumors. Most benign tumors can be differentiated by the patient’s symptoms on presentation and x-ray characteristics.
Osteoid osteomas are most commonly located in the cortices of long bones, occur most frequently in the second decade of life, and will have a distinct nidus surrounded by sclerotic bone on imaging. Osteoid osteomas also have a distinct pain pattern of nocturnal pain that improves with NSAIDS.
Aneurysmal bone cysts (ABC) frequently occur in the spine (25%) and long bones (20%). X-ray findings include a radiolucent, lytic lesion with an intact rim of cortex and bony septae or a “bubbly appearance”. Patients will present with pain, swelling, and if large enough, with a pathologic fracture. MRI will show fluid filled levels that help make the diagnosis of an ABC.
Non-ossifying fibromas are the most common benign tumor in childhood. NOF’s are usually found incidentally on x-ray as they are asymptomatic lesions. However, large lesions may present as pathological fractures. X-rays characteristics include an eccentric “bubbly” lytic lesion with a sclerotic outer rim.
Unicameral bone cysts are benign lesions that are often asymptomatic and found incidentally on x-ray. X-rays will show a unilocular or septated lucency with thinning of the outer cortex. MRI characteristics include dark T1 images, bright T2 images, and rim enhancement with gadolinium contrast.
Giant cell tumors (GCT) are locally aggressive benign tumors that typically present with pain, swelling, and joint stiffness. GCT’s most commonly occur in adults age 20 to 40 with the distal femur and proximal tibia being the most common sites. X-rays characteristics typically include an eccentric lytic lesion involving the epiphysis and metaphysis. GCT’s can also extend through subchondral bone into the joint and cortical bone reaching the surrounding soft tissue.
References
Lewis VO, Morris CD, Parsons TW. Malignant and benign bone tumors you are likely to see. AAOS Instructional Course Lecture 2013; 62: 535- 549.
UBC. http://www.wheelessonine.com. Accessed 5/23/2015.
Incorrect
Osteoid osteomas, aneurysmal bone cysts, non-ossfiying fibromas, unicameral bone cysts, and giant cell tumors are all benign bone tumors. Most benign tumors can be differentiated by the patient’s symptoms on presentation and x-ray characteristics.
Osteoid osteomas are most commonly located in the cortices of long bones, occur most frequently in the second decade of life, and will have a distinct nidus surrounded by sclerotic bone on imaging. Osteoid osteomas also have a distinct pain pattern of nocturnal pain that improves with NSAIDS.
Aneurysmal bone cysts (ABC) frequently occur in the spine (25%) and long bones (20%). X-ray findings include a radiolucent, lytic lesion with an intact rim of cortex and bony septae or a “bubbly appearance”. Patients will present with pain, swelling, and if large enough, with a pathologic fracture. MRI will show fluid filled levels that help make the diagnosis of an ABC.
Non-ossifying fibromas are the most common benign tumor in childhood. NOF’s are usually found incidentally on x-ray as they are asymptomatic lesions. However, large lesions may present as pathological fractures. X-rays characteristics include an eccentric “bubbly” lytic lesion with a sclerotic outer rim.
Unicameral bone cysts are benign lesions that are often asymptomatic and found incidentally on x-ray. X-rays will show a unilocular or septated lucency with thinning of the outer cortex. MRI characteristics include dark T1 images, bright T2 images, and rim enhancement with gadolinium contrast.
Giant cell tumors (GCT) are locally aggressive benign tumors that typically present with pain, swelling, and joint stiffness. GCT’s most commonly occur in adults age 20 to 40 with the distal femur and proximal tibia being the most common sites. X-rays characteristics typically include an eccentric lytic lesion involving the epiphysis and metaphysis. GCT’s can also extend through subchondral bone into the joint and cortical bone reaching the surrounding soft tissue.
References
Lewis VO, Morris CD, Parsons TW. Malignant and benign bone tumors you are likely to see. AAOS Instructional Course Lecture 2013; 62: 535- 549.
UBC. http://www.wheelessonine.com. Accessed 5/23/2015.
-
Question 9 of 21
9. Question
A 54-year old male presents with a one month history of left hip pain. He is an auto-mechanic and slipped on a patch of wet flooring. He was able to catch himself from falling but in doing so felt a sharp pain in the lateral aspect of his left hip. He continued to work after the injury despite having significant pain and limitations secondary to the injury. AP x-ray of the left hip obtained in the office reveals a fracture of the greater trochanter. He denies any history of cancer, weight loss, or fatigue. MRI shows a lytic osseous lesion within the left greater trochanter and extending to the surrounding soft tissue. What is the next best step in treating this patient?
Correct
CT scans with contrast of the chest, abdomen, and pelvis were ordered to determine the primary source of the patient’s metastases of unknown origin. A bone scan was also ordered to determine if other bones were involved. CT scans revealed left and right adrenal masses, an enlarged right kidney, and multiple pulmonary nodules consistent with renal cell carcinoma with metastatic disease to the lungs and adrenal glands.
Any new, destructive bone lesion found in a patient over 40 years old should be evaluated for metastatic disease, myeloma, and lymphoma first. The most likely cause of a malignant appearing bone mass in patients over 40 years old is metastatic disease, with lung metastases being the most common source. The pneumonic “bacon, lettuce, tomatoe, kosher, and pickle” is often used to remember the most common sources of metastatic disease “breast, lung, thyroid, kidney, and prostate”. The most common site of bony metastases is the thoracic spine and most common site of pathological fracture resulting from metastases is the proximal femur. Thyroid and prostate metastases tend to be the least aggressive and slow growing with a median survival rate of 48 and 40 months, respectively. Survival rates for breast cancer metastases can vary widely but on average is around 24 months. Patients with kidney and lung metastases have the poorest prognosis with mean survival rates as low as 6 months.
The work-up of a solitary bone lesion requires a detailed history and physical exam. Patients may report a history of prior cancer, bowel or bladder changes, or weight loss. Physical exam of the breasts, skin, chest, prostate, and neck/thyroid is important. Physical exam may pick lymphadenopathy, costovertebral tenderness, abdominal masses or breast masses that may help identify the primary source. Neurological deficits may indicate spinal cord or nerve root compression from a metastatic lesion.
Initial lab work-up should include a CBC with differential, ESR, CRP, complete metabolic panel, urinalysis, and co-agulation studies. Additional lab work may include a serum protein electrophoresis if multiple myeloma is suspected, a serum carcinoembryonic antigen if colon or pancreatic cancer is suspected, and a serum cancer antigen 125 if there is a risk of ovarian cancer.
Initial imaging work-up for a solitary bone lesion of unknown etiology includes AP and lateral x-rays of the affected limb. Plain x-rays provide the best diagnostic value of any other imaging. CT scan with contrast of the chest, abdomen, and pelvis helps identifying the primary source and can be used in staging the patient’s disease. A bone scan is important in the work-up of a solitary tumor to help differentiate a primary bone tumor vs. multiple lesions from metastatic disease. PET/CT measures metabolic activity such as blood flow, oxygen use, and glucose metabolism to pinpoint the location of abnormal activity produced by a cancerous tumor. PET/CT is more specific for metastatic disease than bone scan. PET/CT is also used to monitor response to therapy. A bone survey, or skeletal survey, is a series of x-rays of the major bones in the body including the skull, ribs, spine, pelvis, and long bones. A skeletal survey is a useful test to evaluate bony involvement in patients with known multiple myeloma as bone scans may be falsely negative in 30% of these patients.Many orthopedic providers have different thresholds for when to refer patients and when to continue with further work-up of a solitary bone lesion of unknown etiology. If a primary malignant bone tumor is considered in the differential, referral to an orthopedic oncologist is recommended. However, in patients over 40 years old with a solitary bone lesion, a metastatic carcinoma is approximately 500 times more likely than a primary bone sarcoma. A thorough work-up can help avoid inappropriate referral and expedite treatment, which improves patient care.
References
Bueker PJ, Rothrock CP. Current Management of Skeletal Metastasis. Orthopedic knowledge Online Journal. 2010 (8)8. Accesses September 1, 2010.
Metastatic Cancer of Bone. http://www.orthobullets.com. Accessed June 8, 2015.
Incorrect
CT scans with contrast of the chest, abdomen, and pelvis were ordered to determine the primary source of the patient’s metastases of unknown origin. A bone scan was also ordered to determine if other bones were involved. CT scans revealed left and right adrenal masses, an enlarged right kidney, and multiple pulmonary nodules consistent with renal cell carcinoma with metastatic disease to the lungs and adrenal glands.
Any new, destructive bone lesion found in a patient over 40 years old should be evaluated for metastatic disease, myeloma, and lymphoma first. The most likely cause of a malignant appearing bone mass in patients over 40 years old is metastatic disease, with lung metastases being the most common source. The pneumonic “bacon, lettuce, tomatoe, kosher, and pickle” is often used to remember the most common sources of metastatic disease “breast, lung, thyroid, kidney, and prostate”. The most common site of bony metastases is the thoracic spine and most common site of pathological fracture resulting from metastases is the proximal femur. Thyroid and prostate metastases tend to be the least aggressive and slow growing with a median survival rate of 48 and 40 months, respectively. Survival rates for breast cancer metastases can vary widely but on average is around 24 months. Patients with kidney and lung metastases have the poorest prognosis with mean survival rates as low as 6 months.
The work-up of a solitary bone lesion requires a detailed history and physical exam. Patients may report a history of prior cancer, bowel or bladder changes, or weight loss. Physical exam of the breasts, skin, chest, prostate, and neck/thyroid is important. Physical exam may pick lymphadenopathy, costovertebral tenderness, abdominal masses or breast masses that may help identify the primary source. Neurological deficits may indicate spinal cord or nerve root compression from a metastatic lesion.
Initial lab work-up should include a CBC with differential, ESR, CRP, complete metabolic panel, urinalysis, and co-agulation studies. Additional lab work may include a serum protein electrophoresis if multiple myeloma is suspected, a serum carcinoembryonic antigen if colon or pancreatic cancer is suspected, and a serum cancer antigen 125 if there is a risk of ovarian cancer.
Initial imaging work-up for a solitary bone lesion of unknown etiology includes AP and lateral x-rays of the affected limb. Plain x-rays provide the best diagnostic value of any other imaging. CT scan with contrast of the chest, abdomen, and pelvis helps identifying the primary source and can be used in staging the patient’s disease. A bone scan is important in the work-up of a solitary tumor to help differentiate a primary bone tumor vs. multiple lesions from metastatic disease. PET/CT measures metabolic activity such as blood flow, oxygen use, and glucose metabolism to pinpoint the location of abnormal activity produced by a cancerous tumor. PET/CT is more specific for metastatic disease than bone scan. PET/CT is also used to monitor response to therapy. A bone survey, or skeletal survey, is a series of x-rays of the major bones in the body including the skull, ribs, spine, pelvis, and long bones. A skeletal survey is a useful test to evaluate bony involvement in patients with known multiple myeloma as bone scans may be falsely negative in 30% of these patients.Many orthopedic providers have different thresholds for when to refer patients and when to continue with further work-up of a solitary bone lesion of unknown etiology. If a primary malignant bone tumor is considered in the differential, referral to an orthopedic oncologist is recommended. However, in patients over 40 years old with a solitary bone lesion, a metastatic carcinoma is approximately 500 times more likely than a primary bone sarcoma. A thorough work-up can help avoid inappropriate referral and expedite treatment, which improves patient care.
References
Bueker PJ, Rothrock CP. Current Management of Skeletal Metastasis. Orthopedic knowledge Online Journal. 2010 (8)8. Accesses September 1, 2010.
Metastatic Cancer of Bone. http://www.orthobullets.com. Accessed June 8, 2015.
-
Question 10 of 21
10. Question
A 47-year old female presents to the emergency department with severe right hip pain after a standing height fall at home. She took a few steps off her front porch and her right leg collapsed from underneath her. She was unable to bear weight and was brought to the ED by EMS. X-rays of the right femur reveal a pathological subtrochanteric femur fracture. The patient was diagnosed with small cell lung cancer in the right upper lobe 6 months ago and is currently undergoing chemotherapy treatments. PET/CT scan one month ago revealed invasion into the mediastinum and right paravertebral area. You are called to evaluate the patient in the ED. Which choice is the next best step in treating this patient?
Correct
The primary source of metastatic lesions should be agreed upon by the patient’s team of healthcare providers prior to surgical fixation. This team often includes a radiologist familiar with identifying bony lesions, the patient’s oncologist who may have already identified the primary source, and the orthopedic surgeon who may obtain a biopsy for frozen section prior to fixation. In most cases of suspected metastatic disease, open biopsy with intraoperative frozen section is performed to confirm the primary source. A definitive diagnosis should be made prior to proceeding with fixation in order to avoid potential spread of an unknown primary malignant bone lesion. However, patients with advanced breast, lung, or prostate cancer that has spread to other organs do not require further work-up of a new bone lesion, including a confirmatory biopsy, prior to fixation. One exception to this is that any new lesion in an area previously treated with radiation therapy should be biopsied to rule out radiation induced sarcoma. A preoperative percutaneous core needle biopsy is recommended if a malignant sarcoma is suspected.
Resection and endoprosthetic reconstruction is the treatment of choice for pathological fractures of the femoral neck. Total hip arthroplasty is recommended if the acetabulum is involved. Hemiarthroplasty is more stable than total hip arthroplasty and can be used for isolated femoral neck fractures. Subtrochanteric and femoral shaft fractures, and impending fractures, should be treated with a long intramedullary nail that spans the length of the femur.
External beam radiation therapy is performed postoperatively once the wound heals to decrease pain and minimize disease progression. Postoperative radiation exposure has no effect on callus formation and healing rates. Tumors that are radiosensitive, or respond to lower doses of radiation include myeloma, lymphoma, breast, and prostate carcinomas. Lung and thyroid carcinomas, along with melanoma, have intermediate responsiveness to radiation therapy. Renal cell carcinomas and sarcomas are the least responsive and require higher doses of radiation.
Bisphophonate therapy use postoperatively inhibits growth of tumor cells, decreases metastatic spread, provides pain relief, and decreases recurrent fracture rates. Bisphophonates also help treat hypercalcemia associated with bone destruction from the malignancy.References
Quinn RW, Randall RL, Benvenia J, Berven SH, Raskin KA. Contemporary Management of Metastatic Bone Disease: Tips and tools of the trade for general practitioners. Instructional Course Lecture 2014; 63: 431-441.
Metastatic Disease. http://www.orthbullets.com. Accessed 6/22/15.
Incorrect
The primary source of metastatic lesions should be agreed upon by the patient’s team of healthcare providers prior to surgical fixation. This team often includes a radiologist familiar with identifying bony lesions, the patient’s oncologist who may have already identified the primary source, and the orthopedic surgeon who may obtain a biopsy for frozen section prior to fixation. In most cases of suspected metastatic disease, open biopsy with intraoperative frozen section is performed to confirm the primary source. A definitive diagnosis should be made prior to proceeding with fixation in order to avoid potential spread of an unknown primary malignant bone lesion. However, patients with advanced breast, lung, or prostate cancer that has spread to other organs do not require further work-up of a new bone lesion, including a confirmatory biopsy, prior to fixation. One exception to this is that any new lesion in an area previously treated with radiation therapy should be biopsied to rule out radiation induced sarcoma. A preoperative percutaneous core needle biopsy is recommended if a malignant sarcoma is suspected.
Resection and endoprosthetic reconstruction is the treatment of choice for pathological fractures of the femoral neck. Total hip arthroplasty is recommended if the acetabulum is involved. Hemiarthroplasty is more stable than total hip arthroplasty and can be used for isolated femoral neck fractures. Subtrochanteric and femoral shaft fractures, and impending fractures, should be treated with a long intramedullary nail that spans the length of the femur.
External beam radiation therapy is performed postoperatively once the wound heals to decrease pain and minimize disease progression. Postoperative radiation exposure has no effect on callus formation and healing rates. Tumors that are radiosensitive, or respond to lower doses of radiation include myeloma, lymphoma, breast, and prostate carcinomas. Lung and thyroid carcinomas, along with melanoma, have intermediate responsiveness to radiation therapy. Renal cell carcinomas and sarcomas are the least responsive and require higher doses of radiation.
Bisphophonate therapy use postoperatively inhibits growth of tumor cells, decreases metastatic spread, provides pain relief, and decreases recurrent fracture rates. Bisphophonates also help treat hypercalcemia associated with bone destruction from the malignancy.References
Quinn RW, Randall RL, Benvenia J, Berven SH, Raskin KA. Contemporary Management of Metastatic Bone Disease: Tips and tools of the trade for general practitioners. Instructional Course Lecture 2014; 63: 431-441.
Metastatic Disease. http://www.orthbullets.com. Accessed 6/22/15.
-
Question 11 of 21
11. Question
A 77-year old male presents to your office with a one month history of atraumatic left thigh pain. The patient is followed by a local oncologist who diagnosed him with multiple myeloma 2 years ago. He has known metastatic lesions throughout his spine. Recent x-rays and MRI confirm the source of the patient’s thigh pain is an impending fracture from a metastatic lesion. The patient’s life expectancy is greater than 6 months. What is the best next step in treatment of this patient’s impending fracture?
Correct
The patient has a metastatic lesion with multiple myeloma as the known primary source. X-ray and MRI findings are consistent with a metastatic lesion to the femoral shaft. The lesion involves over 80% of the lateral cortex and the associated pain is interfering with the patient’s ability to ambulate. Intramedullary fixation is recommended to stabilize the femur for ambulation and to improve his quality of life.
In general, impending fractures of the lower extremity in patients expected to live greater than 6 to 12 weeks should be stabilized with surgical fixation. However, not all pathological lesions should be treated operatively with surgical stabilization. Surgery is reserved for patients at a high risk of fracture in order to prevent comorbidities associated with a sudden fracture. Surgical stabilization prior to fracture prevents soft tissue injury which improves post-operative rehabilitation.
Predicting fracture risk helps determine if surgical fixation is necessary in an elective setting. The most widely used rating system to predict pathological fractures is the Mirels’ rating system. Mirels uses a scoring system based on four tumor characteristics to help guide treatment. The four characteristics include site of the lesion, nature of the lesion, size of the lesion, and pain. Each characteristic is broken into three categories and assigned a score for each. The tumor characteristic, categories, and corresponding scores are as follows. Site of lesion (1. Upper extremity, 2. Lower extremity, 3. Peritrochanteric) , nature of lesion (1. Blastic, 2. Mixed, 3. Lytic), size of the lesion expressed as a fraction of cortical thickness (1. <1/3, 2. 1/3 to 2/3, 3. > 2/3), and pain (1. Mild, 2. Moderate, 3. Functional). Prophylactic intramedullary fixation is recommended for a lesion with an overall score >9 and should be considered with a score of 8. A lesion with an overall score of 7 or less can be treated nonoperatively with radiation and chemotherapy. Surgical decision making is ultimately up to the treating orthopedic surgeon and may also be influenced by patient size and activity level.This patient has a lesion in the lower extremity (scores 2), a lytic lesion on x-ray (scores 3), over 2/3 cortical disruption of the lateral cortex (scores 3), and functional pain (scores 3). Overall the patient has a Mirels’ rating score of an 11, which would recommend prophylactic intramedullary nailing.
References
Jawad MU, Scully SP. In Brief: Classifications in Breif: Mirel’s Classification: Metastatic disease in long bones and impending pathological fractures. Clin Orthop Relat Res. 2010. Oct; 468 (10): 2825-2827.
Quinn RW, Randall RL, Benvenia J, Berven SH, Raskin KA. Contemporary Management of Metastatic Bone Disease: Tips and tools of the trade for general practitioners. Instructional Course Lecture 2014; 63: 431-441.
Incorrect
The patient has a metastatic lesion with multiple myeloma as the known primary source. X-ray and MRI findings are consistent with a metastatic lesion to the femoral shaft. The lesion involves over 80% of the lateral cortex and the associated pain is interfering with the patient’s ability to ambulate. Intramedullary fixation is recommended to stabilize the femur for ambulation and to improve his quality of life.
In general, impending fractures of the lower extremity in patients expected to live greater than 6 to 12 weeks should be stabilized with surgical fixation. However, not all pathological lesions should be treated operatively with surgical stabilization. Surgery is reserved for patients at a high risk of fracture in order to prevent comorbidities associated with a sudden fracture. Surgical stabilization prior to fracture prevents soft tissue injury which improves post-operative rehabilitation.
Predicting fracture risk helps determine if surgical fixation is necessary in an elective setting. The most widely used rating system to predict pathological fractures is the Mirels’ rating system. Mirels uses a scoring system based on four tumor characteristics to help guide treatment. The four characteristics include site of the lesion, nature of the lesion, size of the lesion, and pain. Each characteristic is broken into three categories and assigned a score for each. The tumor characteristic, categories, and corresponding scores are as follows. Site of lesion (1. Upper extremity, 2. Lower extremity, 3. Peritrochanteric) , nature of lesion (1. Blastic, 2. Mixed, 3. Lytic), size of the lesion expressed as a fraction of cortical thickness (1. <1/3, 2. 1/3 to 2/3, 3. > 2/3), and pain (1. Mild, 2. Moderate, 3. Functional). Prophylactic intramedullary fixation is recommended for a lesion with an overall score >9 and should be considered with a score of 8. A lesion with an overall score of 7 or less can be treated nonoperatively with radiation and chemotherapy. Surgical decision making is ultimately up to the treating orthopedic surgeon and may also be influenced by patient size and activity level.This patient has a lesion in the lower extremity (scores 2), a lytic lesion on x-ray (scores 3), over 2/3 cortical disruption of the lateral cortex (scores 3), and functional pain (scores 3). Overall the patient has a Mirels’ rating score of an 11, which would recommend prophylactic intramedullary nailing.
References
Jawad MU, Scully SP. In Brief: Classifications in Breif: Mirel’s Classification: Metastatic disease in long bones and impending pathological fractures. Clin Orthop Relat Res. 2010. Oct; 468 (10): 2825-2827.
Quinn RW, Randall RL, Benvenia J, Berven SH, Raskin KA. Contemporary Management of Metastatic Bone Disease: Tips and tools of the trade for general practitioners. Instructional Course Lecture 2014; 63: 431-441.
-
Question 12 of 21
12. Question
A 17-year old male presents to the office with a mass on the lateral aspect of his right knee. He noticed the mass approximately 6 months ago and doesn’t believe it has grown since. He denies pain at rest but he is starting to have pain over the mass when running. On exam the patient has a firm, nontender 2 cm mass over the lateral supracondylar femur. A positive Ober’s test is noted. AP and lateral x-rays are shown above. What is the most likely diagnosis?
Correct
The patient has an osteochondroma on the lateral supracondylar femur that is causing iliotibial band (ITB) tendinitis.
Osteochondroma, or osteocartilaginous exostosis, is a benign cartilaginous tumor that develops during childhood or adolescence. The tumor most commonly arises from the physes of the distal femur, proximal tibia, and proximal humerus but may occur at other sites. The tumor is essentially part of the physis that separates and continues to grow independently. Osteochodroma is the most common bone tumor accounting for approximately 40% of all benign bone tumors. The tumor usually presents as a painless mass arising from the surface of bone. Large tumors can cause pain by mechanical impingement and neurovascular compression. Osteochondromas will enlarge during skeletal growth and become latent during skeletal maturity.
Multiple hereditary exostoses is an autosomal dominant condition characterized by multiple osteochondromas throughout the skeleton. Almost all patients with the condition will have developed multiple osteochondromas by the age of 12 and new lesions rarely develop after skeletal maturity. Patients with multiple exostoses may present with angular deformities in bone, growth disturbances, and limb length discrepancies. Multiple sites of exostoses have a higher rate of malignant transformation to chondrosarcoma as does persistence of greater than a 2 cm cartilage cap after skeletal maturity. A solitary osteochondroma has a 1% percent chance of malignant transformation compared to a 10% chance in patients with multiple hereditary exostoses.
X-rays alone can make the diagnosis of an osteochondroma. X-rays will reveal an exostosis that may have a sessile (flattened) or pedunculated (stalk-like) appearance. The cortex of the lesion is continuous with the cortex of the native bone. The mass has a thin 2-3 mm cartilaginous cap that may be visualized on x-ray as irregular calcifications. MRI is the best image modality to measure the cartilage cap and asses for potential malignant transformation.Treatment includes observation in asymptomatic patients. Operative treatment may be indicated in lesions that cause pain, mechanical impingement, deformity, or neurovascular compression. Operative treatment includes a marginal resection that includes the base of the stalk, the cartilage cap, and overlying periosteum. Cartilaginous growth of over 1.5 cm in a dormant osteochondroma or after skeletal maturity may signify malignant transformation and surgical resection should be performed. The reoccurrence rate after marginal excision of a solitary exostosis is less than 5%.
Chondrosarcoma is a malignant cartilage lesion more common in males (male to female ratio is 1.5 to 1) in their 40s to 80s. There are multiple subtypes with a variety of presentations making it difficult to diagnose and treat. Chondrosarcomas are generally found in the metaphysis of long bones and typically present with a history of constant, progressive pain not relieved with rest.Enchondroma is a solitary, benign, intramedullary cartilage tumor, often occurring in the short tubular bones of the hands, feet, or proximal long bones. Typically, this tumor causes no symptoms and is found incidentally on x-rays taken for another complaint. They usually begin and grow in childhood; peak incidence is in the third decade of life and equal between males and females. Incidence for transformation to a malignant tumor is rare (1%).
Osteoid osteoma is a benign bone tumor, usually in the proximal end of long bones that can affect people of all ages. Male to female ratio is 2-3 to 1. They are small (< 2 cm) but may have surrounding reactive bone formation. Typical presentation is a moderate intensity, dull, aching, pain which may or may not respond to NSAIDS. Most will disappear after a few years.
Chondroblastoma is a rare, benign bone tumor derived from chondroblasts. Most commonly found in the epiphysis of long bones around the knee, hip, shoulder, or elbow. Although it can affect anybody, 80% occur by the age of 25 and males are twice as likely as females to get chondroblastomas. The most common symptom is pain, but may include joint stiffness, muscle atrophy, or a limp. A chondroblastoma does have potential to metastasize.
References
Arkader A. Clinical and Surgical Approach to Benign Bone Tumors in Children. Orthopedic Knowledge Online Journal. 1/1/12.
Osteochondroma. http://www.bonetumor.org. Accessed on 7/13/15.
Osteochondroma. http://www.wheelesonline.com. Accessed 7/14/15.
Incorrect
The patient has an osteochondroma on the lateral supracondylar femur that is causing iliotibial band (ITB) tendinitis.
Osteochondroma, or osteocartilaginous exostosis, is a benign cartilaginous tumor that develops during childhood or adolescence. The tumor most commonly arises from the physes of the distal femur, proximal tibia, and proximal humerus but may occur at other sites. The tumor is essentially part of the physis that separates and continues to grow independently. Osteochodroma is the most common bone tumor accounting for approximately 40% of all benign bone tumors. The tumor usually presents as a painless mass arising from the surface of bone. Large tumors can cause pain by mechanical impingement and neurovascular compression. Osteochondromas will enlarge during skeletal growth and become latent during skeletal maturity.
Multiple hereditary exostoses is an autosomal dominant condition characterized by multiple osteochondromas throughout the skeleton. Almost all patients with the condition will have developed multiple osteochondromas by the age of 12 and new lesions rarely develop after skeletal maturity. Patients with multiple exostoses may present with angular deformities in bone, growth disturbances, and limb length discrepancies. Multiple sites of exostoses have a higher rate of malignant transformation to chondrosarcoma as does persistence of greater than a 2 cm cartilage cap after skeletal maturity. A solitary osteochondroma has a 1% percent chance of malignant transformation compared to a 10% chance in patients with multiple hereditary exostoses.
X-rays alone can make the diagnosis of an osteochondroma. X-rays will reveal an exostosis that may have a sessile (flattened) or pedunculated (stalk-like) appearance. The cortex of the lesion is continuous with the cortex of the native bone. The mass has a thin 2-3 mm cartilaginous cap that may be visualized on x-ray as irregular calcifications. MRI is the best image modality to measure the cartilage cap and asses for potential malignant transformation.Treatment includes observation in asymptomatic patients. Operative treatment may be indicated in lesions that cause pain, mechanical impingement, deformity, or neurovascular compression. Operative treatment includes a marginal resection that includes the base of the stalk, the cartilage cap, and overlying periosteum. Cartilaginous growth of over 1.5 cm in a dormant osteochondroma or after skeletal maturity may signify malignant transformation and surgical resection should be performed. The reoccurrence rate after marginal excision of a solitary exostosis is less than 5%.
Chondrosarcoma is a malignant cartilage lesion more common in males (male to female ratio is 1.5 to 1) in their 40s to 80s. There are multiple subtypes with a variety of presentations making it difficult to diagnose and treat. Chondrosarcomas are generally found in the metaphysis of long bones and typically present with a history of constant, progressive pain not relieved with rest.Enchondroma is a solitary, benign, intramedullary cartilage tumor, often occurring in the short tubular bones of the hands, feet, or proximal long bones. Typically, this tumor causes no symptoms and is found incidentally on x-rays taken for another complaint. They usually begin and grow in childhood; peak incidence is in the third decade of life and equal between males and females. Incidence for transformation to a malignant tumor is rare (1%).
Osteoid osteoma is a benign bone tumor, usually in the proximal end of long bones that can affect people of all ages. Male to female ratio is 2-3 to 1. They are small (< 2 cm) but may have surrounding reactive bone formation. Typical presentation is a moderate intensity, dull, aching, pain which may or may not respond to NSAIDS. Most will disappear after a few years.
Chondroblastoma is a rare, benign bone tumor derived from chondroblasts. Most commonly found in the epiphysis of long bones around the knee, hip, shoulder, or elbow. Although it can affect anybody, 80% occur by the age of 25 and males are twice as likely as females to get chondroblastomas. The most common symptom is pain, but may include joint stiffness, muscle atrophy, or a limp. A chondroblastoma does have potential to metastasize.
References
Arkader A. Clinical and Surgical Approach to Benign Bone Tumors in Children. Orthopedic Knowledge Online Journal. 1/1/12.
Osteochondroma. http://www.bonetumor.org. Accessed on 7/13/15.
Osteochondroma. http://www.wheelesonline.com. Accessed 7/14/15.
-
Question 13 of 21
13. Question
A 42-year old male presents with a 2-month history of right sided groin pain. He denies injury or any known precipitating event. The pain is constant at rest and seems to be worse at night. He denies any history of known cancer. An AP pelvis x-ray, coronal MRI image, and an axial CT image are shown above. Results from a needle biopsy are consistent with a low grade chondrosarcoma. What is the next best step in treatment?
Correct
Chondrosarcoma is the third most common malignant bone tumor behind myeloma and osteosarcoma. Although rare in presentation, an estimated 600 patients are diagnosed each year in the United States. Chondrosarcomas typically occur in flat bones including the shoulder and pelvic girdles, but may also occur in the extremities. Common presentation includes males between the ages of 40-60 years of age with deep pain that is persistent at rest. Chondrosarcomas can appear radiographically as a low grade intracompartmental lesion much like an enchondroma. However, serial radiographs done each month will show continued growth with chondrosarcomas.
Chondrosarcomas come in two forms: primary and secondary. Primary chondrosarcomas arise de novo from the intramedullary space of bone. Secondary chondrosarcomas arises from preexisting benign lesions such as osteochondromas, multiple hereditary exostosis, enchondromas, Ollier’s disease, and Mafucci’s syndrome. Typically, 85% of chondrosarcomas are grade 1 or 2.
On plain x-rays it can be difficult to differentiate between benign lesions and chondrosarcomas. X-rays will likely show a lytic, lobular lesion with a pattern of calcification within a cartilaginous matrix. Low grade lesions may show cortical thickening and endosteal erosion where high-grade lesions show cortical destruction and soft tissue invasion. CT scan is sensitive in determining the extent of cortical destruction and the presence of new lucencies which suggest malignant transformation. CT scan of the chest may be done as high grade tumors are prone to pulmonary metasteses. MRI may be used to determine the extent of soft tissue involvement.
Generally chondrosarcoma is a slow growing malignancy but tumor aggressiveness can vary. Radiographic appearance and histiologic findings are not great predictors of tumor aggression. Factors that suggest a high grade malignancy include increasing size of the tumor, the presence of pain that persists at rest, and age over 40. Lesions located in the pelvis and proximal extremities are more likely to be malignant than tumors in the distal extremities.
The diagnosis is often influenced by clinical history and physical exam findings as radiographic and histiologic findings are unreliable in confirming the diagnosis. Pain, inflammation, and increasing tumor signs suggest malignancy. Serial radiographs to monitor changes of tumor size over time may be the most accurate method in determining malignant potential. A definitive diagnosis is established by correlating clinical and radiographic findings, gross pathological findings, and histopathological findings. A biopsy may be performed prior to the definitive surgical procedure. However, biopsies poorly differentiate low grade vs. high grade lesions and therefore are not usually helpful in preoperative planning. Special care is taken during the biopsy to avoid spreading tumors cells along the needle tract.
Treatment is often dictated by the tumor grade which is based on histiologic findings, anatomic location, and the presence or absence of metastasis. Radiation and chemotherapy treatments are ineffective due to the slow growing nature and poor vascularity of chondrosarcomas. Surgical intervention including intralesional curettage or wide surgical excision is the treatment of choice for all chondrosarcomas. Intralesional curettage is primarily used in low grade lesions in the extremities but is not recommended for chondrosarcomas of the pelvis due to the higher recurrence rates. Wide surgical excision involves resection of the involved bone and surrounding tissue with the goal of removing all malignant cells. During a wide surgical excision procedure, intraoperative tissue is sent for frozen section until all margins are free of malignant cells. If tumor cells are noted on the marginal excision tissue then the margins are widened during the same operative setting until all margins are negative.
References
Scharschmidt T, Mayerson J. Chondrosarcoma. Orthopedic Knowledge Online Journal 2010. 8(10)Patel SR, Benjamin RS. Soft Tissue and Bone Sarcomas and Bone Metastases. In: Kasper D, Fauci A, Hauser S, Longo D, Jameson J, Loscalzo J. eds. Harrison’s Principles of Internal Medicine, 19e. New York, NY: McGraw-Hill; 2015.
Incorrect
Chondrosarcoma is the third most common malignant bone tumor behind myeloma and osteosarcoma. Although rare in presentation, an estimated 600 patients are diagnosed each year in the United States. Chondrosarcomas typically occur in flat bones including the shoulder and pelvic girdles, but may also occur in the extremities. Common presentation includes males between the ages of 40-60 years of age with deep pain that is persistent at rest. Chondrosarcomas can appear radiographically as a low grade intracompartmental lesion much like an enchondroma. However, serial radiographs done each month will show continued growth with chondrosarcomas.
Chondrosarcomas come in two forms: primary and secondary. Primary chondrosarcomas arise de novo from the intramedullary space of bone. Secondary chondrosarcomas arises from preexisting benign lesions such as osteochondromas, multiple hereditary exostosis, enchondromas, Ollier’s disease, and Mafucci’s syndrome. Typically, 85% of chondrosarcomas are grade 1 or 2.
On plain x-rays it can be difficult to differentiate between benign lesions and chondrosarcomas. X-rays will likely show a lytic, lobular lesion with a pattern of calcification within a cartilaginous matrix. Low grade lesions may show cortical thickening and endosteal erosion where high-grade lesions show cortical destruction and soft tissue invasion. CT scan is sensitive in determining the extent of cortical destruction and the presence of new lucencies which suggest malignant transformation. CT scan of the chest may be done as high grade tumors are prone to pulmonary metasteses. MRI may be used to determine the extent of soft tissue involvement.
Generally chondrosarcoma is a slow growing malignancy but tumor aggressiveness can vary. Radiographic appearance and histiologic findings are not great predictors of tumor aggression. Factors that suggest a high grade malignancy include increasing size of the tumor, the presence of pain that persists at rest, and age over 40. Lesions located in the pelvis and proximal extremities are more likely to be malignant than tumors in the distal extremities.
The diagnosis is often influenced by clinical history and physical exam findings as radiographic and histiologic findings are unreliable in confirming the diagnosis. Pain, inflammation, and increasing tumor signs suggest malignancy. Serial radiographs to monitor changes of tumor size over time may be the most accurate method in determining malignant potential. A definitive diagnosis is established by correlating clinical and radiographic findings, gross pathological findings, and histopathological findings. A biopsy may be performed prior to the definitive surgical procedure. However, biopsies poorly differentiate low grade vs. high grade lesions and therefore are not usually helpful in preoperative planning. Special care is taken during the biopsy to avoid spreading tumors cells along the needle tract.
Treatment is often dictated by the tumor grade which is based on histiologic findings, anatomic location, and the presence or absence of metastasis. Radiation and chemotherapy treatments are ineffective due to the slow growing nature and poor vascularity of chondrosarcomas. Surgical intervention including intralesional curettage or wide surgical excision is the treatment of choice for all chondrosarcomas. Intralesional curettage is primarily used in low grade lesions in the extremities but is not recommended for chondrosarcomas of the pelvis due to the higher recurrence rates. Wide surgical excision involves resection of the involved bone and surrounding tissue with the goal of removing all malignant cells. During a wide surgical excision procedure, intraoperative tissue is sent for frozen section until all margins are free of malignant cells. If tumor cells are noted on the marginal excision tissue then the margins are widened during the same operative setting until all margins are negative.
References
Scharschmidt T, Mayerson J. Chondrosarcoma. Orthopedic Knowledge Online Journal 2010. 8(10)Patel SR, Benjamin RS. Soft Tissue and Bone Sarcomas and Bone Metastases. In: Kasper D, Fauci A, Hauser S, Longo D, Jameson J, Loscalzo J. eds. Harrison’s Principles of Internal Medicine, 19e. New York, NY: McGraw-Hill; 2015.
-
Question 14 of 21
14. Question
A 20-year old male presents with a 10 month history of progressive right knee pain and soft tissue swelling. AP x-ray and coronal MRI images are shown above. What is the most appropriate treatment plan?
Correct
Osteosarcoma is the most common primary malignant bone tumor affecting approximately 560 children and adolescents each year in the United States. The prevalence has a bimodal distribution with the majority occurring in the second decade of life and a second peak occurring in elderly patients with Paget’s disease. Classic osteosarcoma occurs during the adolescent growth spurt on the metaphyseal areas of long bones. The distal femur is the most common site with the proximal tibia and proximal humerus being the 2nd and 3rd most common sites, respectively.
Patients typically present with weeks to months of progressive pain in the involved extremity. The pain is persistent during the day and night. Physical exam findings may include decrease range of motion, erythema, and a palpable mass. AP and lateral x-rays of the involved extremity should be done first. X-rays may show lytic destruction of metaphyseal bone with eventual cortical breakthrough. A Codman triangle, or a triangular area of new periosteal bone formation, may be noted at the diaphyseal end of the tumor. The tumor may expand into the soft tissue creating a sunburst pattern of new bone outside the involved bone. MRI of the entire involved bone should be performed if an osteosarcoma is present on x-ray. MRI determines the extent of soft issue involvement, neurovascular involvement, and the presence of a “skip lesion” or a separate tumor found proximally. MRI also helps surgical planning to determine location of safe margins and if a limb-sparing procedure is possible.
Once an osteosarcoma is suspected on x-rays a biopsy is performed before the definitive surgical procedure. The biopsy site is chosen from a location that will be removed during the definitive tumor resection. Osteosarcoma has several subtypes with varying characteristics including tumor aggressiveness, location, and prognosis. Intramedullary forms of osteosarcoma have four subtypes including conventional (most common type accounting for 80% of all osteosarcomas), telangiectatic, low-grade, and small cell. Surface osteosarcomas are divided into four subtypes including parosteal, periosteal, and high-grade.
Once the diagnosis of osteosarcoma is confirmed with imaging and biopsy, the tumor is staged based on factors such as tumor size, lymph node involvement, and distal metastases. A CT scan of the chest should be done in all patients with an osteosarcoma as the lungs are the most common site of metastases. Treatment typically involves preoperative chemotherapy, wide surgical resection, and postoperative chemotherapy. Limp sparing surgery whenever possible is the standard approach. Amputation may be necessary if the wide resection results in a non functioning limb. Approximately 70% of patients without metastases will be disease free after chemotherapy and wide resection surgery. When a metastatic lesion is present the 10 year survival rate drops below 30%.
References
Messerschmitt PJ, Garcia RM, Abdul-Karim FW, Greenfield EM, Getty PJ. Osteosarcoma. JAAOS 2009; 17: 515-527.
Osteosarcoma. http://www.wheelessonline.com. Accessed 9/20/2015.
Incorrect
Osteosarcoma is the most common primary malignant bone tumor affecting approximately 560 children and adolescents each year in the United States. The prevalence has a bimodal distribution with the majority occurring in the second decade of life and a second peak occurring in elderly patients with Paget’s disease. Classic osteosarcoma occurs during the adolescent growth spurt on the metaphyseal areas of long bones. The distal femur is the most common site with the proximal tibia and proximal humerus being the 2nd and 3rd most common sites, respectively.
Patients typically present with weeks to months of progressive pain in the involved extremity. The pain is persistent during the day and night. Physical exam findings may include decrease range of motion, erythema, and a palpable mass. AP and lateral x-rays of the involved extremity should be done first. X-rays may show lytic destruction of metaphyseal bone with eventual cortical breakthrough. A Codman triangle, or a triangular area of new periosteal bone formation, may be noted at the diaphyseal end of the tumor. The tumor may expand into the soft tissue creating a sunburst pattern of new bone outside the involved bone. MRI of the entire involved bone should be performed if an osteosarcoma is present on x-ray. MRI determines the extent of soft issue involvement, neurovascular involvement, and the presence of a “skip lesion” or a separate tumor found proximally. MRI also helps surgical planning to determine location of safe margins and if a limb-sparing procedure is possible.
Once an osteosarcoma is suspected on x-rays a biopsy is performed before the definitive surgical procedure. The biopsy site is chosen from a location that will be removed during the definitive tumor resection. Osteosarcoma has several subtypes with varying characteristics including tumor aggressiveness, location, and prognosis. Intramedullary forms of osteosarcoma have four subtypes including conventional (most common type accounting for 80% of all osteosarcomas), telangiectatic, low-grade, and small cell. Surface osteosarcomas are divided into four subtypes including parosteal, periosteal, and high-grade.
Once the diagnosis of osteosarcoma is confirmed with imaging and biopsy, the tumor is staged based on factors such as tumor size, lymph node involvement, and distal metastases. A CT scan of the chest should be done in all patients with an osteosarcoma as the lungs are the most common site of metastases. Treatment typically involves preoperative chemotherapy, wide surgical resection, and postoperative chemotherapy. Limp sparing surgery whenever possible is the standard approach. Amputation may be necessary if the wide resection results in a non functioning limb. Approximately 70% of patients without metastases will be disease free after chemotherapy and wide resection surgery. When a metastatic lesion is present the 10 year survival rate drops below 30%.
References
Messerschmitt PJ, Garcia RM, Abdul-Karim FW, Greenfield EM, Getty PJ. Osteosarcoma. JAAOS 2009; 17: 515-527.
Osteosarcoma. http://www.wheelessonline.com. Accessed 9/20/2015.
-
Question 15 of 21
15. Question
A 15-year old male presents to the primary care office with complaints of right arm pain. He states that he has been experiencing the pain for a few months and really didn’t think about it until he started baseball practice recently. Upon further questioning, you find that he also admits to recent weight loss of 10lbs and a low grade fever. What is the most likely diagnosis?
Correct
Ewing Sarcoma is a diffuse endothelioma of bone, accounting for only 10% of primary malignant bone tumors. In most cases, it is found in male patients between the ages of 5 and 25. Although the pelvis is the most common location, it can be found in any bone in the body, including the femur, tibia, humerus, and scapula. The most common presenting complaint is pain at the site of the tumor. Weight loss, appetite changes, and fever are associated symptoms. On x-ray, the appearance shows as a central lytic tumor of the diaphyseal-metaphyseal bone. It takes on an onion skin appearance due to permeative destruction of cortical bone. Another feature that can be found on x-ray is the reactive hair-on-end appearance made by bone forming along the periosteal vessels that run between the cortex and the elevated periosteum. An MRI should be obtained to document the extent of local disease. Biopsy is necessary for definitive diagnosis. This is an aggressive malignancy with high rates of metastases and local recurrence. In addition to surgical excision of the lesion, chemotherapy is recommended for one year. The 5-year survival rate of a primary tumor that is resectable is about 70%. The most common sites for metastases are the lung, bone (especially the spine) and bone marrow. The survival rate for patients who present with a nonlocalized disease is about 15-25%.
References
Randall R, Ward R, Hoang BH. Chapter 5. Musculoskeletal Oncology. In: Skinner HB, McMahon PJ. eds. Current Diagnosis & Treatment in Orthopedics, 5e. New York, NY: McGraw-Hill; 2014. http://accessmedicine.mhmedical.com/content.aspx?bookid=675&Sectionid=45451711. Accessed September 04, 2015.Graham DK, Craddock JA, Quinones RR, Keating AK, Maloney K, Foreman NK, Giller RH, Greffe BS. Neoplastic Disease. In: Hay WW, Jr., Levin MJ, Deterding RR, Abzug MJ. eds. CURRENT Diagnosis & Treatment: Pediatrics, 22e. New York, NY: McGraw-Hill; 2013. http://accessmedicine.mhmedical.com/content.aspx?bookid=1016&Sectionid=61604063. Accessed September 04, 2015.
Incorrect
Ewing Sarcoma is a diffuse endothelioma of bone, accounting for only 10% of primary malignant bone tumors. In most cases, it is found in male patients between the ages of 5 and 25. Although the pelvis is the most common location, it can be found in any bone in the body, including the femur, tibia, humerus, and scapula. The most common presenting complaint is pain at the site of the tumor. Weight loss, appetite changes, and fever are associated symptoms. On x-ray, the appearance shows as a central lytic tumor of the diaphyseal-metaphyseal bone. It takes on an onion skin appearance due to permeative destruction of cortical bone. Another feature that can be found on x-ray is the reactive hair-on-end appearance made by bone forming along the periosteal vessels that run between the cortex and the elevated periosteum. An MRI should be obtained to document the extent of local disease. Biopsy is necessary for definitive diagnosis. This is an aggressive malignancy with high rates of metastases and local recurrence. In addition to surgical excision of the lesion, chemotherapy is recommended for one year. The 5-year survival rate of a primary tumor that is resectable is about 70%. The most common sites for metastases are the lung, bone (especially the spine) and bone marrow. The survival rate for patients who present with a nonlocalized disease is about 15-25%.
References
Randall R, Ward R, Hoang BH. Chapter 5. Musculoskeletal Oncology. In: Skinner HB, McMahon PJ. eds. Current Diagnosis & Treatment in Orthopedics, 5e. New York, NY: McGraw-Hill; 2014. http://accessmedicine.mhmedical.com/content.aspx?bookid=675&Sectionid=45451711. Accessed September 04, 2015.Graham DK, Craddock JA, Quinones RR, Keating AK, Maloney K, Foreman NK, Giller RH, Greffe BS. Neoplastic Disease. In: Hay WW, Jr., Levin MJ, Deterding RR, Abzug MJ. eds. CURRENT Diagnosis & Treatment: Pediatrics, 22e. New York, NY: McGraw-Hill; 2013. http://accessmedicine.mhmedical.com/content.aspx?bookid=1016&Sectionid=61604063. Accessed September 04, 2015.
-
Question 16 of 21
16. Question
A 17-year old male presents with progressive pain in his left shin for 3 months. Swelling is noted in the left calf and he has difficultly bearing weight on the leg. AP x-ray of the left leg, axial MRI, and axial CT images are shown above. What is the most likely diagnosis?
Correct
An aneurysmal bone cyst (ABC) is a benign expansile lesion consisting of multiple blood filled cavities separated by bony septae. They commonly present in long bones including the femur and tibia but may occur in other locations such as the pelvis, vertebral body, and posterior elements of the spine. Typical presentation includes males under 20 years of age who complain of pain and swelling at the affected site. ABC’s in the spine (25% of all ABC’s) may present with neurological deficits.
ABC’s are classified by tumor aggression as growth rate can be variable. Low grade tumors may be well contained, centrally located, and have limited expansion. More aggressive ABC’s can expand quickly around or through cortical bone. X-ray characteristics include a radiolucent lesion arising from the medullary canal or metaphysis. The tumor can expand through cortical bone but typically has a thin rim of new bone surrounding the lesion. A “bubbly-appearance” created by bony septation is seen within the lesion. CT scan helps define the outer cortex and can aid in planning for the biopsy. MRI will best show multiple fluid filled septations with characteristic fluid-fluid levels created by layering of blood and serum.
ABC’s may arise or grow adjacent to other bone tumors such as giant cell tumors, fibrous dysplasia, non-ossifying fibroma, chondroblastoma, osteoblastoma, and osteosarcoma. These secondary ABC’s represent 30% of all lesions. Differentiating ABC’s from other similar presenting bone tumors can be difficult. A simple bone cyst is centrally located and does not expand through the cortex. Giant cell tumors rarely occur in patients under 20 years old and typically begin at the epiphysis and extend to the metaphysis. Symptoms of fever, weight loss, and fatigue are associated with malignancies and should not be present with benign tumors like ABC’s.
Incisional biopsy is the standard approach for histological evaluation to confirm the diagnosis. Multiple samples must be taken to rule out other tumors that demonstrate an aneurysmal component such as giant cell tumors, telangiectatic osteosarcoma, and fracture through a simple cyst. Fine needle biopsy alone has an unacceptably high missed diagnosis rate.
Treatment consists of aggressive intralesional curettage and bone grafting once the definitive diagnosis has been established. However, curettage and bone grafting alone has recurrence rates of up to 20-40%. Using adjunct therapies, including phenol, polymethylmethacrylate cement, liquid nitrogen, radionuclide ablation, and/or high speed burr, can significantly lower recurrence rates. Wide excision with en bloc resection has the lowest recurrence rates but often times will negatively affect the involved extremities function. ABC’s that present with acute fracture should be allowed to heal before definitive treatment is performed.
References
Rapp TB, Ward JP, Alaia MJ. Aneurysmal Bone Cyst. JAAOS 2012. 20: 233-241.
Aneurysmal Bone Cyst. http://www.orthobullets.com. Accessed on 9/24/14.
Incorrect
An aneurysmal bone cyst (ABC) is a benign expansile lesion consisting of multiple blood filled cavities separated by bony septae. They commonly present in long bones including the femur and tibia but may occur in other locations such as the pelvis, vertebral body, and posterior elements of the spine. Typical presentation includes males under 20 years of age who complain of pain and swelling at the affected site. ABC’s in the spine (25% of all ABC’s) may present with neurological deficits.
ABC’s are classified by tumor aggression as growth rate can be variable. Low grade tumors may be well contained, centrally located, and have limited expansion. More aggressive ABC’s can expand quickly around or through cortical bone. X-ray characteristics include a radiolucent lesion arising from the medullary canal or metaphysis. The tumor can expand through cortical bone but typically has a thin rim of new bone surrounding the lesion. A “bubbly-appearance” created by bony septation is seen within the lesion. CT scan helps define the outer cortex and can aid in planning for the biopsy. MRI will best show multiple fluid filled septations with characteristic fluid-fluid levels created by layering of blood and serum.
ABC’s may arise or grow adjacent to other bone tumors such as giant cell tumors, fibrous dysplasia, non-ossifying fibroma, chondroblastoma, osteoblastoma, and osteosarcoma. These secondary ABC’s represent 30% of all lesions. Differentiating ABC’s from other similar presenting bone tumors can be difficult. A simple bone cyst is centrally located and does not expand through the cortex. Giant cell tumors rarely occur in patients under 20 years old and typically begin at the epiphysis and extend to the metaphysis. Symptoms of fever, weight loss, and fatigue are associated with malignancies and should not be present with benign tumors like ABC’s.
Incisional biopsy is the standard approach for histological evaluation to confirm the diagnosis. Multiple samples must be taken to rule out other tumors that demonstrate an aneurysmal component such as giant cell tumors, telangiectatic osteosarcoma, and fracture through a simple cyst. Fine needle biopsy alone has an unacceptably high missed diagnosis rate.
Treatment consists of aggressive intralesional curettage and bone grafting once the definitive diagnosis has been established. However, curettage and bone grafting alone has recurrence rates of up to 20-40%. Using adjunct therapies, including phenol, polymethylmethacrylate cement, liquid nitrogen, radionuclide ablation, and/or high speed burr, can significantly lower recurrence rates. Wide excision with en bloc resection has the lowest recurrence rates but often times will negatively affect the involved extremities function. ABC’s that present with acute fracture should be allowed to heal before definitive treatment is performed.
References
Rapp TB, Ward JP, Alaia MJ. Aneurysmal Bone Cyst. JAAOS 2012. 20: 233-241.
Aneurysmal Bone Cyst. http://www.orthobullets.com. Accessed on 9/24/14.
-
Question 17 of 21
17. Question
A 41-year old female presents with progressive right knee pain for the past 4 months. X-ray, MRI, and CT images are shown above. What is the most likely diagnosis?Correct
Giant cell tumors (GCT’s) are benign but locally aggressive tumors that typically involve the epiphysis and adjacent metaphysis of long bones. The distal femur is the most common location (over 50% occur around the knee) followed by the proximal tibia then the distal radius. Unlike most bone tumors, giant cell tumors are more prevalent in females between the ages of 30-50 years old.
Giant cell tumors typically present as a solitary tumor that may enlarge and erode through the subchondral bone, articular cartilage, and ligaments. However, expansion through the articular cartilage is rare. Symptoms may include pain, decrease range of motion of the affected joint, and a palpable mass on exam. Pain is worse with weight bearing and up to 12% of patients with GCT’s will present with pathological fracture.
GCT’s are classified into three grades: grade 1 (4%) are considered inactive and are not locally aggressive, grade 2 (74%) are locally aggressive tumors that expand into and thin the cortices, grade 3 (22%) are locally aggressive, expand through the cortices, and may have soft tissue involvement.
Radiographically, GCT’s can appear malignant in nature with bony destruction and cortical breakthrough. MRI is helpful in differentiating commonly confused bone tumors such as aneurysmal bone cysts, brown tumor of hyperparathyroidism, chondroblastoma, osteosarcoma, and metastatic disease. Typical MRI findings include low intensity on T1-weighted images, homogeneous in nature, and well defined boarders without surrounding bone and soft tissue edema. CT scan is generally ordered to aid in choosing the biopsy location and determine degree of cortical thinning. Absence of intralesional calcification is consistent with GCT’s. A biopsy is necessary to make a definitive diagnosis. Histology results show classic multinucleated giant cells distributed evenly throughout the lesion.
GCT’s may cause metastases to the lungs, lymph nodes, or undergo malignant transformation. Metastases to the lungs and lymph nodes behave the same way as the benign primary tumor. Very rarely do GCT’s transform to a malignant sarcoma and transformation is thought to be caused by prior radiation exposure.
Treatment is often dictated by the tumor grade. The goal of treatment is to remove all tumor cells while still trying to preserve the integrity of the involved joint. Curettage alone results in unacceptable recurrence rates of up to 34%. Adjunct therapies are often used after curettage including phenol, cement, and liquid nitrogen in an attempt to further decrease recurrence rates. With adjunct therapies, recurrence rates drop to 3%. Once the tumor is removed the void in the bone may be filled with cement or bone graft for structural support. Grade 3 tumors may require en bloc excision with wide margins. This may result in the need for a large osteoarticular allograft, joint reconstruction procedure, or amputation depending on the function of the limb after excision. Patients should be followed closely after treatment with serial radiographs of the involved site and chest to rule out pulmonary involvement. Recurrence rates have been reported between 3 to 25% and GCT’s have been found to reoccur as far out as 12 years after treatment.
ReferencesRandall R, Ward R, Hoang BH. Chapter 5. Musculoskeletal Oncology. In: Skinner HB, McMahon PJ. eds. Current Diagnosis & Treatment in Orthopedics, 5e. New York, NY: McGraw-Hill; 2014. http://accessmedicine.mhmedical.com/content.aspx?bookid=675&Sectionid=45451711. Accessed September 04, 2015.
Lewis VO, Aboulafia AJ. Giant Cell Tumor of Bone. Orthopedic Online Journal 2007. 5 (5).
Incorrect
Giant cell tumors (GCT’s) are benign but locally aggressive tumors that typically involve the epiphysis and adjacent metaphysis of long bones. The distal femur is the most common location (over 50% occur around the knee) followed by the proximal tibia then the distal radius. Unlike most bone tumors, giant cell tumors are more prevalent in females between the ages of 30-50 years old.
Giant cell tumors typically present as a solitary tumor that may enlarge and erode through the subchondral bone, articular cartilage, and ligaments. However, expansion through the articular cartilage is rare. Symptoms may include pain, decrease range of motion of the affected joint, and a palpable mass on exam. Pain is worse with weight bearing and up to 12% of patients with GCT’s will present with pathological fracture.
GCT’s are classified into three grades: grade 1 (4%) are considered inactive and are not locally aggressive, grade 2 (74%) are locally aggressive tumors that expand into and thin the cortices, grade 3 (22%) are locally aggressive, expand through the cortices, and may have soft tissue involvement.
Radiographically, GCT’s can appear malignant in nature with bony destruction and cortical breakthrough. MRI is helpful in differentiating commonly confused bone tumors such as aneurysmal bone cysts, brown tumor of hyperparathyroidism, chondroblastoma, osteosarcoma, and metastatic disease. Typical MRI findings include low intensity on T1-weighted images, homogeneous in nature, and well defined boarders without surrounding bone and soft tissue edema. CT scan is generally ordered to aid in choosing the biopsy location and determine degree of cortical thinning. Absence of intralesional calcification is consistent with GCT’s. A biopsy is necessary to make a definitive diagnosis. Histology results show classic multinucleated giant cells distributed evenly throughout the lesion.
GCT’s may cause metastases to the lungs, lymph nodes, or undergo malignant transformation. Metastases to the lungs and lymph nodes behave the same way as the benign primary tumor. Very rarely do GCT’s transform to a malignant sarcoma and transformation is thought to be caused by prior radiation exposure.
Treatment is often dictated by the tumor grade. The goal of treatment is to remove all tumor cells while still trying to preserve the integrity of the involved joint. Curettage alone results in unacceptable recurrence rates of up to 34%. Adjunct therapies are often used after curettage including phenol, cement, and liquid nitrogen in an attempt to further decrease recurrence rates. With adjunct therapies, recurrence rates drop to 3%. Once the tumor is removed the void in the bone may be filled with cement or bone graft for structural support. Grade 3 tumors may require en bloc excision with wide margins. This may result in the need for a large osteoarticular allograft, joint reconstruction procedure, or amputation depending on the function of the limb after excision. Patients should be followed closely after treatment with serial radiographs of the involved site and chest to rule out pulmonary involvement. Recurrence rates have been reported between 3 to 25% and GCT’s have been found to reoccur as far out as 12 years after treatment.
ReferencesRandall R, Ward R, Hoang BH. Chapter 5. Musculoskeletal Oncology. In: Skinner HB, McMahon PJ. eds. Current Diagnosis & Treatment in Orthopedics, 5e. New York, NY: McGraw-Hill; 2014. http://accessmedicine.mhmedical.com/content.aspx?bookid=675&Sectionid=45451711. Accessed September 04, 2015.
Lewis VO, Aboulafia AJ. Giant Cell Tumor of Bone. Orthopedic Online Journal 2007. 5 (5).
-
Question 18 of 21
18. Question
A 13-year old male presents with left ankle pain after a fall 5 days ago. X-rays done at an outside urgent care facility show no evidence of a fracture (Figures 1 and 2 above). His exam reveals pain to palpation over the distal fibula physis consistent with a Salter-Harris type 1 fracture. An incidental bone cyst is noted in the distal tibia and the patient’s parents would like to know if the lesion requires further treatment. Which choice is the most appropriate recommendation?
Correct
The lesion appears benign on x-ray as the zone of transition between the lesion and adjacent normal bone has a well defined margin. There is no periosteal reaction or wide zone of transition that would suggest a malignancy. The lesion is also asymptomatic, found incidentally on x-ray, and occurred in a skeletally immature patient which is typical of benign lesions. Common benign metaphyseal tumors include non-ossifying fibroma (NOF) and bone cysts (solitary, aneurysmal, and giant cell). Characteristic findings on x-ray of a non-ossifying fibroma (NOF) include a sharply demarcated, asymmetrical, lucent lesion with a thin sclerotic rim. NOF’s are commonly found in the metaphyseal region adjacent to the physis and migrate to the diaphysis with skeletal growth. X-rays are diagnostic of NOF’s and MRI is not routinely ordered. Biopsy is not indicated if the lesion appears benign and the patient has no associated pain. NOF’s will progressively ossify during skeletal maturity and rarely need any treatment or follow-up. In large lesions that occupy more than 50% of the diameter of bone prophylactic curettage and bone grafting may be indicated to prevent pathological fracture. The patient’s parents were advised to have the x-rays repeated after 6 months to make sure the lesion didn’t enlarge or change in appearance.
References
Bone Tumors. http://www.radiologyassistant.nl . Accessed 10/18/2015
Temple HT, Scully SP, Aboulafia AJ. Benign bone tumors. AAOS Instructional Course Lectures 2002. Volume 51, 429-439.
Incorrect
The lesion appears benign on x-ray as the zone of transition between the lesion and adjacent normal bone has a well defined margin. There is no periosteal reaction or wide zone of transition that would suggest a malignancy. The lesion is also asymptomatic, found incidentally on x-ray, and occurred in a skeletally immature patient which is typical of benign lesions. Common benign metaphyseal tumors include non-ossifying fibroma (NOF) and bone cysts (solitary, aneurysmal, and giant cell). Characteristic findings on x-ray of a non-ossifying fibroma (NOF) include a sharply demarcated, asymmetrical, lucent lesion with a thin sclerotic rim. NOF’s are commonly found in the metaphyseal region adjacent to the physis and migrate to the diaphysis with skeletal growth. X-rays are diagnostic of NOF’s and MRI is not routinely ordered. Biopsy is not indicated if the lesion appears benign and the patient has no associated pain. NOF’s will progressively ossify during skeletal maturity and rarely need any treatment or follow-up. In large lesions that occupy more than 50% of the diameter of bone prophylactic curettage and bone grafting may be indicated to prevent pathological fracture. The patient’s parents were advised to have the x-rays repeated after 6 months to make sure the lesion didn’t enlarge or change in appearance.
References
Bone Tumors. http://www.radiologyassistant.nl . Accessed 10/18/2015
Temple HT, Scully SP, Aboulafia AJ. Benign bone tumors. AAOS Instructional Course Lectures 2002. Volume 51, 429-439.
-
Question 19 of 21
19. Question
A 63-year old female with a history of stage 4 non-small cell lung cancer presents to the emergency department with increasing pain in the right femur. The pain started 3 months ago and she denies any known injury or precipitating event. The pain is present with activities and at rest. A recent CT scan showed metastatic disease to her liver, adrenal glands, and pelvis. A bone scan showed increase uptake in bilateral femurs. X-rays (shown above) and CT of the right femur show cortical erosion along the diaphysis of the midshaft femur with periosteal reaction consistent with metastatic disease. The lesion involves approximately 20% of the medial femoral cortex. What is the best treatment choice for this patient?
Correct
Metastatic disease is the most common cause of a destructive bone lesion in adults. Nearly 80% of all patients with solid tumors will develop symptomatic metastases to the spine, pelvis, or extremities. Tumors that are frequently associated with skeletal metastases include multiple myeloma, prostate cancer, breast cancer, lung cancer, thyroid cancer, and renal cancer. The goals of treatment for patients with metastatic disease to the bone include pain control, prevention of pathological fracture, prevention of tumor progression, and preservation of function. Several factors are considered when deciding the most appropriate treatment strategy including patient choice, life expectancy, risk of pathological fracture, and patient’s functional status. Surgical stabilization may be indicated when patients present with a complete or impending fracture. Impending fracture risk is often determined by the presence of functional pain, site of involvement, and > 50% destruction of cortical bone.
Surgical stabilization is not indicated in this case as there is < 50% cortical destruction and the pain is present at rest and with activities. It’s important to note that not all long bone metastatic lesions will result in pathological fracture; only 10-30 percent of patients with a long bone lesion will experience a pathological fracture.
External beam radiation therapy (EBRT) is a standard treatment for painful skeletal metastases. EBRT delivers high dose radiation from outside the body to a focused area of tumor tissue. EBRT kills the tumor cells by damaging the DNA with radiation which helps prevent tumor progression. Local field EBRT can provide pain relief in up to 80% of patients, with complete relief in up to 30%. Pain relief has been shown to last up to 6 months in at least 50% of treatments. EBRT is often administered in one dose or a single fraction of radiation. If surgical stabilization is performed, EBRT is routinely administered postoperatively. Often palliative care patients that receive EBRT have short life expectancies so the reoccurrence rate after treatment is infrequent. Bisphosphonates are often used as an adjunct therapy for patients with metastatic disease. Bisphosphonate therapy has been shown to reduce the risk of skeletal related events including pathological fracture, spinal cord compression, and malignant hypercalcemia.
The patient has widespread metastatic disease and a life expectancy is predictably short. Therefore the primary goal of treatment is pain control. The patient underwent external beam radiation therapy, chemotherapy, and IV bisphosphonate treatments. She was discharged from the hospital to a hospice house for palliative care.
References
1. Yu M, Hoffe SE. Overview of the epidemiology, clinical presentation, diagnosis, and management of adult patients with metastasis. UpToDate. Accessed on 1/18/15.
2. Coleman R. The management of bone metastases. The Oncologist. Accessed 1/18/15.
3. Metastatic disease. http://www.orthobullets.com. Accessed on 1/18/15.
Incorrect
Metastatic disease is the most common cause of a destructive bone lesion in adults. Nearly 80% of all patients with solid tumors will develop symptomatic metastases to the spine, pelvis, or extremities. Tumors that are frequently associated with skeletal metastases include multiple myeloma, prostate cancer, breast cancer, lung cancer, thyroid cancer, and renal cancer. The goals of treatment for patients with metastatic disease to the bone include pain control, prevention of pathological fracture, prevention of tumor progression, and preservation of function. Several factors are considered when deciding the most appropriate treatment strategy including patient choice, life expectancy, risk of pathological fracture, and patient’s functional status. Surgical stabilization may be indicated when patients present with a complete or impending fracture. Impending fracture risk is often determined by the presence of functional pain, site of involvement, and > 50% destruction of cortical bone.
Surgical stabilization is not indicated in this case as there is < 50% cortical destruction and the pain is present at rest and with activities. It’s important to note that not all long bone metastatic lesions will result in pathological fracture; only 10-30 percent of patients with a long bone lesion will experience a pathological fracture.
External beam radiation therapy (EBRT) is a standard treatment for painful skeletal metastases. EBRT delivers high dose radiation from outside the body to a focused area of tumor tissue. EBRT kills the tumor cells by damaging the DNA with radiation which helps prevent tumor progression. Local field EBRT can provide pain relief in up to 80% of patients, with complete relief in up to 30%. Pain relief has been shown to last up to 6 months in at least 50% of treatments. EBRT is often administered in one dose or a single fraction of radiation. If surgical stabilization is performed, EBRT is routinely administered postoperatively. Often palliative care patients that receive EBRT have short life expectancies so the reoccurrence rate after treatment is infrequent. Bisphosphonates are often used as an adjunct therapy for patients with metastatic disease. Bisphosphonate therapy has been shown to reduce the risk of skeletal related events including pathological fracture, spinal cord compression, and malignant hypercalcemia.
The patient has widespread metastatic disease and a life expectancy is predictably short. Therefore the primary goal of treatment is pain control. The patient underwent external beam radiation therapy, chemotherapy, and IV bisphosphonate treatments. She was discharged from the hospital to a hospice house for palliative care.
References
1. Yu M, Hoffe SE. Overview of the epidemiology, clinical presentation, diagnosis, and management of adult patients with metastasis. UpToDate. Accessed on 1/18/15.
2. Coleman R. The management of bone metastases. The Oncologist. Accessed 1/18/15.
3. Metastatic disease. http://www.orthobullets.com. Accessed on 1/18/15.
-
Question 20 of 21
20. Question
A 65-year-old female presents to the orthopedic clinic with a 6-month history of right low back, gluteal, and thigh pain. The patient works as a nurse and has gradually noticed increasing pain in the right leg. She describes no history of injury or fall. She presented to the emergency room 2 months after the onset of pain for radiographs, which were read as negative. She was referred to a physiatrist, who repeated the AP x-ray shown above, which was also read as negative. The pain significantly affected the patient’s daily activities as well as work. The pain is worse with standing and walking activities, and is getting progressively worse throughout the day. She also complains of night time symptoms that regularly wake her from sleep. The physiatrist gave the patient a prescription for NSAIDs, which she states did improve her symptoms slightly, although the pain has returned. CT scan of the femur is shown above. What is the most likely diagnosis?
Correct
Osteoid osteoma is the most common benign bone-forming lesion and was first characterized by Jaffe in 1935. It is the third most common benign bone neoplasm after osteochondroma and non-ossifying fibroma,which most commonly occurs in persons aged 5–25 years, with a male:female ratio of 2:1. Its elective sites are the long bones, especially the femur and tibia, which are involved more than 50% of the time. Depending on their locations within the bone, osteoid osteomas can be distinguished into cortical (70–75% of cases), medullary (25% of the cases), subperiosteal, and intra-articular.
The lesion is usually less than 1.5 cm in diameter and contains a discrete central area known as the nidus, which is surrounded by dense sclerotic bone tissue. The nidus is composed of thin seams of osteoid or woven bone lined with osteoblasts, which represents a process of bone remodeling with osteoblastic activity. The nidus is surrounded by a region of active bone formation that appears as sclerotic dense bone with various patterns of maturation. The nidus has been shown to be highly vascularized and contain abundant nerve fibers. These nerve fibers belong to the osseous nerve supply, which plays an important role in skeletal development and homeostasis. Osteoid osteomas usually present with a significant amount of soft tissue swelling around the area affected, and this may be secondary to the highly vascular nature of the tumor. Swelling may be mediated by production of prostaglandins, which can affect soft tissue and vascular permeability. Prostaglandin levels in the nidus are 100–1,000 times that of normal bone and are thought to be important in the perception of pain. The pain relief associated with NSAIDs has also been attributed to the inhibition of such prostaglandin production.
Typical presentation includes localized pain that is frequently more severe at night. The pain may occasionally be relieved by NSAIDs. Depending on the location of the lesion, patients may also present with gait disturbances, bony deformity, joint stiffness or contracture, growth disturbances, swelling, and limb length discrepancies.In the case of limb length discrepancies, it was found that the involved extremity was typically the longer one.One possible explanation for limb overgrowth in children may be the resulting inflammatory response and associated hyperemia, especially in patients with lesions located near the open growth plate.
Osteoid osteomas in complex sites such as the pelvis, posterior vertebrae, femoral neck, or hind foot can be difficult to diagnose radiographically. The classic finding on conventional radiography is an oval or round lucency (representing the nidus) with or without calcifications, surrounded by bone sclerosis and periosteal bone neoformation to different degrees. Bone scintigraphy demonstrates increased radionuclide uptake by the nidus. CT remains the method of choice to diagnose and locate the osteoid osteoma nidus, especially in complex anatomical sites. The characteristic appearance on a thinly sliced CT scan is of a low-attenuation nidus with central mineralization and varying degrees of sclerosis surrounding the nidus. A recent report recommends the use of non-ionizing imaging modalities in children, although the role of MRI remains controversial. The appearance of the lesion on MRI may be highly variable, and the presence of associated soft tissue changes and bone marrow edema may result in diagnostic errors.
Management is typically begun with salicylates or NSAIDs because of the above-mentioned control of prostaglandins and pain relief. The time until symptoms subside with treatment using just these medications can vary from several months to several years. Surgical management may be warranted if the pain is too severe, the patient is unresponsive to medication, or the patient is unwilling to wait for the symptoms to spontaneously subside.
Until the late 1990s, open excision with removal of the complete nidus was the only surgical option available. This was found to be highly effective, but sometimes the tumor was difficult to identify intra-operatively, and incomplete resection could result in recurrence. This method followed by a prolonged period of weight-bearing restriction if performed in one of the weight-bearing bones.
CT-guided percutaneous resection, a less-invasive method, is performed with CT guidance to minimize the amount of excised bone and ensure that the entire nidus is resected. This is typically an outpatient procedure, but may require an overnight stay at the hospital. It is usually followed by restriction of activities for 4 weeks with toe-touch weight-bearing if performed on one of the weight-bearing bones.RF ablation has been used successfully to treat osteoid osteomas for more than 20 years and is currently the choice of treatment for most lesions. RF ablation is performed with the use of CT guidance, and the entire procedure generally takes 90 min. The RF electrode is inserted with the tip directed toward the nidus and thermal heating is applied for 4–6 min. The success rate for this procedure has been documented as high as 90%, only requires a brief recovery, and has a low complication rate.
Osteoid osteoma can be a challenging diagnosis if the patient does not present with classic symptoms. A differential diagnosis can include chronic osteomyelitis, stress fracture, non-specific synovitis, arthritis, osteoblastoma, fibrous dysplasia, melorheostosis, or Ewing’s sarcoma. Diagnosis can be even more confusing, such as in our patient, with a presentation outside the typical age group and a non-specific appearance on bone scan and MRI. CT scan remains the modality of choice to diagnose the lesions and the characteristic nidus. Patients who appear with the typical throbbing pain that is worse at night and responds to NSAIDs can obtain a straight forward diagnosis. Currently, the treatment of choice remains CT-guided RF ablation, which has the highest success rate with the lowest recovery time and rate of complications.
References
Shankman S, Deasai P, Beltran J.Subperiosteal osteoid osteoma: radiographic and pathologic manifestations. International Skeletal Society.1997;26(8):457-62.
Laurence N, Epelman M, Markowitz R, et al.Osteoid osteomas: a pain in the night diagnosis. Pediatric Radiology. 2012;42(12):1490-1501.
Atesok K, Alman B, Schemitsch E, et al. Osteoid Osteoma and Osteoblastoma. J Am Acad Orthop Surg. 2011;19(11):678-689.
Cerase A, Priolo F.Skeletal benign bone-forming lesions. European Journal of Radiology. 1998;27:S91-S97.
Incorrect
Osteoid osteoma is the most common benign bone-forming lesion and was first characterized by Jaffe in 1935. It is the third most common benign bone neoplasm after osteochondroma and non-ossifying fibroma,which most commonly occurs in persons aged 5–25 years, with a male:female ratio of 2:1. Its elective sites are the long bones, especially the femur and tibia, which are involved more than 50% of the time. Depending on their locations within the bone, osteoid osteomas can be distinguished into cortical (70–75% of cases), medullary (25% of the cases), subperiosteal, and intra-articular.
The lesion is usually less than 1.5 cm in diameter and contains a discrete central area known as the nidus, which is surrounded by dense sclerotic bone tissue. The nidus is composed of thin seams of osteoid or woven bone lined with osteoblasts, which represents a process of bone remodeling with osteoblastic activity. The nidus is surrounded by a region of active bone formation that appears as sclerotic dense bone with various patterns of maturation. The nidus has been shown to be highly vascularized and contain abundant nerve fibers. These nerve fibers belong to the osseous nerve supply, which plays an important role in skeletal development and homeostasis. Osteoid osteomas usually present with a significant amount of soft tissue swelling around the area affected, and this may be secondary to the highly vascular nature of the tumor. Swelling may be mediated by production of prostaglandins, which can affect soft tissue and vascular permeability. Prostaglandin levels in the nidus are 100–1,000 times that of normal bone and are thought to be important in the perception of pain. The pain relief associated with NSAIDs has also been attributed to the inhibition of such prostaglandin production.
Typical presentation includes localized pain that is frequently more severe at night. The pain may occasionally be relieved by NSAIDs. Depending on the location of the lesion, patients may also present with gait disturbances, bony deformity, joint stiffness or contracture, growth disturbances, swelling, and limb length discrepancies.In the case of limb length discrepancies, it was found that the involved extremity was typically the longer one.One possible explanation for limb overgrowth in children may be the resulting inflammatory response and associated hyperemia, especially in patients with lesions located near the open growth plate.
Osteoid osteomas in complex sites such as the pelvis, posterior vertebrae, femoral neck, or hind foot can be difficult to diagnose radiographically. The classic finding on conventional radiography is an oval or round lucency (representing the nidus) with or without calcifications, surrounded by bone sclerosis and periosteal bone neoformation to different degrees. Bone scintigraphy demonstrates increased radionuclide uptake by the nidus. CT remains the method of choice to diagnose and locate the osteoid osteoma nidus, especially in complex anatomical sites. The characteristic appearance on a thinly sliced CT scan is of a low-attenuation nidus with central mineralization and varying degrees of sclerosis surrounding the nidus. A recent report recommends the use of non-ionizing imaging modalities in children, although the role of MRI remains controversial. The appearance of the lesion on MRI may be highly variable, and the presence of associated soft tissue changes and bone marrow edema may result in diagnostic errors.
Management is typically begun with salicylates or NSAIDs because of the above-mentioned control of prostaglandins and pain relief. The time until symptoms subside with treatment using just these medications can vary from several months to several years. Surgical management may be warranted if the pain is too severe, the patient is unresponsive to medication, or the patient is unwilling to wait for the symptoms to spontaneously subside.
Until the late 1990s, open excision with removal of the complete nidus was the only surgical option available. This was found to be highly effective, but sometimes the tumor was difficult to identify intra-operatively, and incomplete resection could result in recurrence. This method followed by a prolonged period of weight-bearing restriction if performed in one of the weight-bearing bones.
CT-guided percutaneous resection, a less-invasive method, is performed with CT guidance to minimize the amount of excised bone and ensure that the entire nidus is resected. This is typically an outpatient procedure, but may require an overnight stay at the hospital. It is usually followed by restriction of activities for 4 weeks with toe-touch weight-bearing if performed on one of the weight-bearing bones.RF ablation has been used successfully to treat osteoid osteomas for more than 20 years and is currently the choice of treatment for most lesions. RF ablation is performed with the use of CT guidance, and the entire procedure generally takes 90 min. The RF electrode is inserted with the tip directed toward the nidus and thermal heating is applied for 4–6 min. The success rate for this procedure has been documented as high as 90%, only requires a brief recovery, and has a low complication rate.
Osteoid osteoma can be a challenging diagnosis if the patient does not present with classic symptoms. A differential diagnosis can include chronic osteomyelitis, stress fracture, non-specific synovitis, arthritis, osteoblastoma, fibrous dysplasia, melorheostosis, or Ewing’s sarcoma. Diagnosis can be even more confusing, such as in our patient, with a presentation outside the typical age group and a non-specific appearance on bone scan and MRI. CT scan remains the modality of choice to diagnose the lesions and the characteristic nidus. Patients who appear with the typical throbbing pain that is worse at night and responds to NSAIDs can obtain a straight forward diagnosis. Currently, the treatment of choice remains CT-guided RF ablation, which has the highest success rate with the lowest recovery time and rate of complications.
References
Shankman S, Deasai P, Beltran J.Subperiosteal osteoid osteoma: radiographic and pathologic manifestations. International Skeletal Society.1997;26(8):457-62.
Laurence N, Epelman M, Markowitz R, et al.Osteoid osteomas: a pain in the night diagnosis. Pediatric Radiology. 2012;42(12):1490-1501.
Atesok K, Alman B, Schemitsch E, et al. Osteoid Osteoma and Osteoblastoma. J Am Acad Orthop Surg. 2011;19(11):678-689.
Cerase A, Priolo F.Skeletal benign bone-forming lesions. European Journal of Radiology. 1998;27:S91-S97.
-
Question 21 of 21
21. Question
A 21 year-old male presents to the office with 4-5 months of right upper arm pain. He works in a warehouse and lifting objects is becoming increasingly painful. The pain became severe at work which prompted him to visit an urgent care. AP and lateral x-rays of the right humerus show a well circumscribed cystic lesion in the proximal humerus. MRI shows a 4 cm intramedullary lesion within the right proximal humeral diaphysis consistent with a unicameral bone cyst. There is endosteal erosion consistent with an impending or early fracture. What is the best treatment option for this patient?Correct
Unicameral bone cysts (UBC’s), or simple bone cysts, are benign fluid filled lesions that are most commonly found in proximal long bones such as the proximal humerus and femur. UBC’s contain a single fluid filled cavity that contains prostaglandins and other enzymes. UBC’s are usually diagnosed incidentally on x-ray although some may present symptomatically as a pathological fracture. The cyst may encroach and thin corticol bone around the cyst which increases fracture risk. Characteristics found on x-ray and MRI help differentiate UBC’s from other cystic lesions including an aneurysmal bone cyst, fibrous dysplasia, enchondroma, and possible malignancy. 1,2
Patients with an asymptomatic UBC found incidentally on x-ray can be managed with observation if the risk of pathological fracture is low. Risk factors for pathological fracture include a cyst taking up >85% of the transverse diameter of the affected bone and cortical thinning with a cyst wall <0.5mm thick. UBC’s often start to decease in size when skeletal growth is complete. In general patients that present with pathological fracture can be treated conservatively with a sling for 4-6 weeks. Pathological fracture may decompress the UBC, however few, or <10% of lesions, will completely fill in with bone. The most popular treatment for active symptomatic UBC’s that have failed conservative treatment includes a percutaneous intra-lesional injection of either a steroid (methylprednisolone) or bone graft. The technique is performed under general anesthesia as a large bore needle such as an 11 gauge is necessary to drain the lesion. Open curettage and bone grafting is a more invasive technique and has equivalent to higher recurrence rates of 35-45%. 1,2
Answer B.
References
1. Kling TJ, Rougraff BT. Treatment of Active Unicameral Bone Cysts with Percutaneous Injections of Demineralized Bone Matrix and Authogenous bone marrow.
2. Unicameral Bone Cysts. http://www.orthobullets.com. Accessed on 5/19/2018.Incorrect
Unicameral bone cysts (UBC’s), or simple bone cysts, are benign fluid filled lesions that are most commonly found in proximal long bones such as the proximal humerus and femur. UBC’s contain a single fluid filled cavity that contains prostaglandins and other enzymes. UBC’s are usually diagnosed incidentally on x-ray although some may present symptomatically as a pathological fracture. The cyst may encroach and thin corticol bone around the cyst which increases fracture risk. Characteristics found on x-ray and MRI help differentiate UBC’s from other cystic lesions including an aneurysmal bone cyst, fibrous dysplasia, enchondroma, and possible malignancy. 1,2
Patients with an asymptomatic UBC found incidentally on x-ray can be managed with observation if the risk of pathological fracture is low. Risk factors for pathological fracture include a cyst taking up >85% of the transverse diameter of the affected bone and cortical thinning with a cyst wall <0.5mm thick. UBC’s often start to decease in size when skeletal growth is complete. In general patients that present with pathological fracture can be treated conservatively with a sling for 4-6 weeks. Pathological fracture may decompress the UBC, however few, or <10% of lesions, will completely fill in with bone. The most popular treatment for active symptomatic UBC’s that have failed conservative treatment includes a percutaneous intra-lesional injection of either a steroid (methylprednisolone) or bone graft. The technique is performed under general anesthesia as a large bore needle such as an 11 gauge is necessary to drain the lesion. Open curettage and bone grafting is a more invasive technique and has equivalent to higher recurrence rates of 35-45%. 1,2
Answer B.
References
1. Kling TJ, Rougraff BT. Treatment of Active Unicameral Bone Cysts with Percutaneous Injections of Demineralized Bone Matrix and Authogenous bone marrow.
2. Unicameral Bone Cysts. http://www.orthobullets.com. Accessed on 5/19/2018.